oral pathology and medicine mcqs

Join Pak Army as a Medical Officer which is also called M-Cadet or GDMO. Those candidates who have degree of MBBS or BDS cany join Pakistan Army when its vacancies come. Degree must be recognized by PM&DC or equally recognized if I is attained from the foreign.

There are many students who have MBBS or BDS degee but they do not know that how can they join Pakistan Army, Pakistan Air Force or Pakistan Navy as an M-Cadet. A candidate has to clear initial intelligence tests and academic tests if they apply for medical officer. There are different subjects from which questions are given in the academic tests in which Oral Pathology and Medicine MCQs also have their importance.

In the following, most important and repeated questions of Oral Pathology and Medicine MCQs are given which are very helpful for the students to become a medical officer through Short Service Regular Commission.

See also: How to Solve Intelligence Tests?

Must see below: 

1.     Pakistan Army Ranks and SalaryClick Here
2.     Pakistan Navy Ranks & SalaryClick Here
3.     Pakistan Air Force Ranks & Salary Click Here

Oral Pathology and Medicine MCQs for Preparation

Multiple bilateral dentigerous cysts are seen in:__ __?

A. Down’s syndrome
B. Maroteaux lamy syndrome
C. Teacher collin syndrome
D. Gorlin Goltz syndrome

COC is now called as:___________?

A. Odontogenic ghost cell tumor
B. Dentinogenic ghost cell tumor
C. Keratcysticodontogenic tumour
D. A & C

Facial nerve paralysis is common with:__ ___?

A. Pleomorphic adenoma
B. Epidermoid carcinoma
C. Warthin’s stumour
D. Lymphoepithelial carcinoma

The most aggressive and destructive cyst is:___ ___?

A. Periapical cyst
B. Dentigerous cyst
C. Globulomaxillary cyst
D. Nasopalatine cyst

Standard treatment of ameloblastoma:

A. Segmental resection with 1 cm of normal bone
B. Enbloc resection
C. Enucleation
D. Enucleation with cauterization

The most common odontogenic cyst is:__ ____?

A. Primordial cyst
B. Dentigerous cyst
C. Radicular cyst
D. Mucocele

Cyst arising from dental lamina:____ ___?

A. Radicular cyst
B. Paradental cyst
C. Eruption cyst
D. Glandular odontogenic cyst

Adenomatoid odontogenic tumour is characterized histologically by:____ __?

A. Polyhedral epithelial cells
B. Tubular / duct like cells
C. Stellate shaped cells
D. Stratified squamous epithelial cells

The Pathogenesis of Periapical Cyst is______?

A. Increased pressure within the cyst
B. Immune mediated bone destruction
C. Proliferation of epithelium
D. None of the above

A six year old child patient has blue-dome shaped swelling in posterior mandibular region, what will be the treatment plan?

A. Reassure the patient without any treatment
B. Excise the lesion
C. Marsupialization
D. Surgical Excision

Pindborg tumor arises from:__ ___?

A. Basal layer of cells
B. Stratum intermedium
C. Stratum corneum
D. Dental lamina
E. Both B & D

A 36 year old man with an asymptomatic swelling in the body of the mandible with radiographic features of radiolucency with radiopaque flecks in suffering from:

A. Odontogenic keratocyst
B. Calcifying epithelial odontogenic tumor (CEOT)
C. Ameloblastoma
D. None of the above

Multiple periapical radiolucencies are seen in:_ __?

A. Jawcyst basal cell Nevus Syndrome
B. Odontogenic keratocyst
C. Cherubism
D. thyroid disorders

Clear cells are commonly seen in which of the following lesions?

A. Pleomorphic
B. Warthins tumor
C. Mucoepidermoid
D. Adenomatoid odontogenic tumor

The epithelium of a dentigerous cyst is:__ ___?

A. 15-20 cell thick
B. 6-10 cell thick
C. 2-4 cell thick
D. 1-2 cell thick

Dentigerous cyst is associated with the following

A. Impacted 3rd molar
B. Impacted supernumerary tooth
C. Odontome
D. All of the above

A patient with ameloblastoma of the jaw can best be treated by:__ ____?

A. Irradiation
B. Excision
C. Enucleation
D. Surgical removal followed by cauterization

After entering radiolucent lesion in a 30 years old man hollow cavity without epithelial lining is seen, the most probable diagnosis is:__________?

A. Aneurysmal bone cyst
B. Static bone cavity
C. Memorrhagic bone cyst
D. Ameloblastoma

Dentigerous cyst is suspected if the follicular space is more than:

A. 2-3 mm
B. 3-4 mm
C. 1-2 mm
D. >5 mm

Compound odontoma shows:___ ____?

A. Mixed tissue of dental origin with no resemblance to tooth structure
B. Numerous tooth like structure with denticles commonly found in maxillary lateral incisors
C. Haphazardly arranged calcified mass
D. All of the above

Destructively invasive locally malignant with rare metastasis, the lesion is:

A. Fibroma
B. Ameloblastoma
C. Papilloma
D. None of the above

Lesions associated with vital tooth?

A. condensing osteitis
B. cementoma
C. Periapical abscess
D. None of the above

Treatment for cementoma?

A. No treatment
B. Pulpectomy
C. Resection of jaw
D. None of the above

A 25 year old male patient reports with bony expansile swelling of the right body of the mandible & mild paresthesia of the right IDN. OPG shows a multi locular radiolucency without root resorption. What would be your choice of next investigation?

A. Excision biopsy
B. Aspiration Cytology
C. CT Scan
D. Pet Bone scan

Ghost (shadow) cells are seen in:__ ___?

A. Amebloblastic fibroodontoma
B. Calcifying odontogenic cyst
C. Compound odontoma
D. All of the above

Adamantinoma is:___ ___?

A. A tumour from embrynomal cells of developing teeth
B. Also known as Amebloblastoma
C. is a complication of dentigerous cyst
D. All of the above

Adenomatold odontogenic tumour is most commonly found in:

A. Anterior mandible
B. Posterior maxilla
C. Anterior maxilla
D. Ramus of mandible

Multiple odontogenic keratocyst are associated with:___ ___?

A. Gardner’s syndrome
B. Gorlin-Goltz syndrome
C. Goldenhar’s syndrome
D. Grinspan syndrome

Which histopathological type of odontogenic keratocyst is commoner, more invasive & has a greater tendency for recurrence ?

A. Orthokeratinised
B. Parakeratinised
C. Non-Keratinised
D. Diskeratinised

A 40 year old woman has meloblastoma, the histomorphologic features will be:___ ___?

A. Peripheral palisading cellular strand with central loose stellate reticulum
B. Peripheral palisading with central stromal retraction artefact
C. Peripheral palisading cellular strand with peripheral loose stellate reticulum
D. Central loose stellate reticulum shows marked nuclear atypia and numerous mitotic

Which of the following is wrong about keratocyst:

A. Haw low recurrence rate
B. Has low protein content
C. High recurrence rate
D. B and C

Each of the following cyst is associated with an impacted tooth except:__ ____?

A. Dentigerous cyst
B. Clacifying epithelial odontogenic cyst
C. Keratocyst
D. Primordial cyst

Keratocyst has all of the following features except:___ ___?

A. It is more common in mandible
B. May be filled with thin straw coloured fluid
C. Low recurrence rate
D. Expansion of bone clinically seen

A multilocular cyst of the jaw is more likely:__ ____?

A. Dental cyst
B. Dentigerous cyst
C. Keratocyst
D. Simple bone cyst

The cyst with highest recurrence rate is:_ ______?

A. Keratocyst
B. Periapical cyst
C. Nasoalveolar cyst
D. Globulamaxilary cyst

The most ideal expianation for recurrence of odontogenic keratocyst is:__ ____?

A. Increased mitotic activity of the epithelial lining
B. Friability of the epithelial lining
C. Presence of satellite cysts or daughter cysts
D. Continued proliferation of rests of dental lamina

Unicentric, non-functional, anatomically benign, clinically persistent tumor is:_ ____?

A. CEOT
B. Enameloma
C. Odontoma
D. Ameloblastoma

Radiographic finding in pindborg tumour is:__ _____?

A. Sun-burst appearance
B. Onion – peel appearance
C. Driven-snow appearance
D. Cherry -blossom appearance

Robinson’s classification of ameloblastoma does not include:__ ____?

A. Multicentric
B. Non-Functional
C. Anatomically benign
D. clinically persistent

Primordial cyst develops:___ ____?

A. In place of missing teeth
B. In teeth in which crown development is completed
C. In periapical region
D. In mandibular body

Odontogenic keratocyst has the following feature:___ __?

A. Occurs due to infection periapically
B. Is developmental in origin
C. Can be treated by aspiration
D. Has low recurrence rate

Dentigerous cyst is likely to cause which neoplasia?

A. Ameloblastoma
B. Adeno carcinoma
C. Fibrosarcoma
D. All of the above

One of them is not a true cyst:__ ___?

A. Nemorrhagic cyst
B. Median palatal
C. Globulomaxillary
D. Nasolabial

Which of the following is the most common lesion of the mandible?

A. Adamantinoma
B. Osteogenic sarcoma
C. Squamous cell carcinoma
D. Osteoclastoma

Basal layer in primordial cyst is arranged in the form of:_ ____?

A. Tennis racket
B. Picket fence
C. Linear
D. Irregular

Nodular growth of alveolus is seen in:___ ___?

A. Paget’s disease
B. Osteomas
C. Cementifying fibroma
D. All of these

Which of the following shows the presence of cholesterol crystals:__ ____?

A. Keratocyst
B. Periodontal cyst
C. Aneurysmal cyst
D. Hemorrhagic cyst

The most common odontogenic tumour which occurs in relationto an unerupted tooth in the anterior maxilla:____________?

A. Odontogenic adenomatoid tumour
B. Odontoma
C. Myxoma
D. Cementifying fibroma

Which of the following is an odontogenic tumor?

A. Arrhenoblastoma
B. Astrocytoma
C. Ameloblastoma
D. Granular cell tumor

Leisegang rings are found in:__ ____?

A. Calcifying epithelial odontogenic cyst
B. Primordial cyst
C. Calcifying epithelial odontogenic tumor
D. Odontoma

Eruption cyst:___ ____?

A. Transforms into dentigerous cyst
B. Regresses after eruption of the tooth
C. Is found in the place of the missing tooth
D. Is a type of dentigerous cyst

Botryoid odontogenic cyst is a variant of:__ _____?

A. Lateral periodontal cyst
B. Apical periodontal cyst
C. Gingival cyst of new born
D. Gingival cysts of adult

Which of the following is a true neoplasm of functional cementoblasts:

A. Periapical cemental dysplasia
B. Familial cemental dysplasia
C. Benign cementoblastoma
D. Hypercementosis

Compound odontoma shows on a radiograph as:______?

A. Supernumerary teeth
B. Radiolucent and radiopaque areas
C. Masses of calcified areas
D. Distinguishable tooth – like structures

Ameloblastoma most frequently occurs in:______?

A. Mandibular moral region
B. Maxillary molar region
C. Mandibular premolar region
D. Maxillary premolar region

Bifid ribs, multiple radiolucent lesions of the jaws multiple basal cell nevi and flax cerebri calcification are found in:______?

A. Basal cell nevus syndrome
B. Sturge weber syndrome
C. Horner syndrome
D. Hereditary internal polyposis

All of the following lesions may be classified as Odontogenic Tumours EXCEPT:

A. Acanthomatous ameloblastoma
B. Branchial cleft cyst
C. Myxoma
D. Simple ameloblastoma

Fish Net pattern is pemphigus vulgaris is seen in which of the following tests?

A. Direct immunofluorescence
B. Tzanck smear
C. FNAC
D. Histopathology

All of the following are inherited disorders of connective tissue EXCEPT:

A. Alport syndrome
B. Ehlers-Danlos syndrome
C. Marfan syndrome
D. McArdle’s disease

Xeroderma pigmentosum is characterized by:__ ____?

A. Autosomal dominant inheritance
B. Inability to repair sunlight induced damage to DNA
C. Irregular accemulation of melanin in the basal cell layer
D. Acanthosis of epithelium with elongation of rete ridges

Oral lesion associated with ulcerative colitis?

A. Lichen planus
B. pyostomatitis vegentanus
C. sarcoidosis
D. Dermatitis herpetiformis

Ehlers Danlos syndrome is_____?

A. Autosomal Dominant
B. Autosomal recessive
C. X-linked Dominant
D. X-Linked recessive

Cafe au lait macules are seen in:_ ____?

A. Von Reklinghausen’s neurofibromatosis
B. Albright’s syndrome and Bloom’s syndrome
C. All of the above
D. None of the above

Butterfly rash is typically seen in:____ ____?

A. Herpes simplex
B. Systemic lupus erythematosus
C. Scleroderma
D. None of the above

Hydropic degeneration of the basal cell of the straturn germinativum is a feature of_______?

A. Leukoplakia
B. Lichen planus
C. Syphilis
D. Pemphigus

All are diseases of skin except:

A. Erythema multiforme
B. Keratosis follicularis
C. Erythema migrans
D. Psoriasis form lesion

Subepithelial vesicles are characteristic all of the following EXCEPT:

A. Bullous pemphigoid
B. Cicatricial pemphigoid
C. Pemphigus
D. Epidermolysis bullosa acquisita

In Cicatrial pemphigoid, which antigen is bound by IgG on the epidermal side of the salt split skin technique:__ ___?

A. XVII collagen
B. Epiligrin
C. Laminin 5
D. BP antigen 1 & 2

Psoralane Ultraviolet A (PUVA) therapy is advised in_____?

A. Pemphigus vulgaris
B. Apthous ulcers
C. Carcinoma in situ
D. ANUG

Grinspan syndrome is associated with:___ ___?

A. Leukoplakia
B. Lichen planus
C. Aphthous ulcer
D. Oral submucous fibrosis

Multiple pulp stones are seen in:____ ___?

A. Down’s syndrome
B. Ehler’s Danlos syndrome
C. Marfan syndrome
D. Apert’s syndrome

Joint erosions are not a feature of:___ ____?

A. Rhematoid arthritis
B. Psoriasis
C. Multicentric reticulohisticytosis
D. Systemis lupus eythematosus

Pathologic calcification is seen in:___ ___?

A. Scleroderma
B. Lichen planus
C. Dystrophic epidermolysis bullosa
D. Lupus erythematosus

Ectodermal dysplasia is:__ ____?

A. Autosomal recessive
B. Autosomal dominant
C. X-linked dominant
D. X-linked recessive

The swollen degenerating epithelial cell due to acantholysis is:__ ____?

A. Anitschow cell
B. Tzanck cell
C. Ghost cell
D. Prickle cell

Histological clefts in lichen planus are______?

A. Civatte bodies
B. Wickham’s Striae
C. Max – Joseph spaces
D. Auspitz’s sign

Which sites are characteristically affected in Stevens-Johnson syndrome?

A. Liver, spleen, pancreas
B. Conjunctive, genitalia, oral mucosa
C. Oral mucosa, lacrimal apparatus, ears
D. Parotid gland, palate, conjunctive

The primary cause of acantholysis in pemphigus vulgaris is______?

A. auto immunity
B. Intercellular oedema
C. intra epithelial oedema
D. chronic alcoholism

A 3-year old patient has extensive vesicles on lip, tongue, oral mucous membrane, After 2-4 days vesicles rupture at followed by pseudomembrane formation and also some dermal lesions seen what will be the diagnosis?

A. Herpetic stomatitis
B. EM
C. ANUG
D. Steven-Johnson syndrome

Which is a degeneration disorder characterized by atrophic changes of the deeper structures (e.g fat, muscle, cartilage & bone) involving one side of the face:

A. Scleroderma
B. Parry Romberg syndrome
C. Miescher’s syndrome
D. peutz-Jeghers syndrome

Steven-Johnson syndrome involves:__ ____?

A. Type I hypersensitivity reactions
B. Type II hypersensitivity reactions
C. Type III hypersensitivity reactions
D. Type IV hypersensitivity reactions

Mucocutaneous lesions associated with neoplasia_____?

A. pemphigus vegentans
B. Parapemphigus
C. Paraneoplastic pemphigus
D. familial benign pemphigus

Formation of multiple pinpoint bleeding spots on scratching the skin is characteristic of:___ ____?

A. pemphigus vulgaris
B. Lupus erythematosus
C. Psoriasis
D. Herpangina

Psoriasis is associated with:___ ____?

A. Geographic tongue
B. Benign median rhomboid glossitis
C. Lupus erythematosus
D. lupus vulgaris

Lupus erythematosus is:__ ____?

A. Reactive lesion
B. Degenerative condition
C. Autoimmune disorder
D. Neoplastic condition

Bullae formation after striking an intact skin/mucosal surface is known as:__ ___?

A. Tinel’s sign
B. Bablnski’s sign
C. Nikolsky’s sign
D. Chovstek’s sign

Primary lesion in lichen planus is:___________?

A. Macule
B. Papule
C. Vesicle
D. Bulla

Oral diagnostic features of scleroderma include all of the following, except:___ ___?

A. A hard and a rigid tongue
B. Widening of the oral aperture
C. Pseudo ankylosis of the T.M joint
D. Difficulty in swallowing

White radiating lines can be observed in case of lesions of:____ ___?

A. Lichen planus
B. Erythema multiforme
C. Pemphigus
D. Leukoplakia

In which of the following disorders a circulating antibody directed to intercellular cementing substance of stratified squamous epithelium is observed:_________?

A. Lichen planus
B. Verrucous vulgaris
C. Bullous pemphigoid
D. Pemphigus vulgari

Wickham’s striae are seen in:_ _____?

A. Lichen planus
B. Leukoplakia
C. Leukoedema
D. Erythema multiformae

Oral ocular and genital lesions are seen in:__ ___?

A. Erythema multiforma
B. Steven Johnson syndrome
C. SLE
D. None of the above

Oral lesions are not seen in:___ __?

A. Psoriasis
B. Pemphigoid
C. Stevens Johnson syndrome
D. Candidiasis

Lichen planus:___ _?

A. Can undergo malignant change
B. Treated only by medication
C. Must be excised
D. Is a idiosyncrasy reaction

Intra-epithelial bulla are found in:__ ____?

A. pemphigus
B. Bullous pemphigoid
C. Bullous lichen planus
D. Pemphigoid

Which of the following are seen in ectodermal dysplasia?

A. Hyperpyrexia
B. Protuberant lips and frontal bossing
C. Defective or absence of sweat glands
D. Any of the above

Fine Needle aspiration biopsy is indicated to diagnose:___ ____?

A. Traumatic ulcer
B. Pemphigus
C. Necrotic pulp
D. Chronic gingivitis

Target lesions are observed in case of:___ ___?

A. Erythema multiforme
B. Lichenplanus
C. Pemphigus vulgaris
D. Psoriasis

A 60 year old has got severe bulla and target lesion Which erythema around halo and genital lesions:____ ___?

A. Stevens Jhonson syndrome
B. Herpes zoster
C. Herpes simplex
D. Herpangina

Erosive lichen planus resembles which of the following:

A. Monilial gingivitis
B. Desquamative gingivitis
C. Herpetic gingivitis
D. Acute ulcerative gingivitis

Immunoflourescence is seen at basement membrane as patchy distribution in:__ ___?

A. Lichen planus
B. Pemphigus
C. Pemphigoid
D. Lupus erythematosus

Immunoflourescence test is positive in:___ ____?

A. Psoriasis
B. pemphigus vulgaris
C. Lupus erythematosus
D. Scleroderma
E. Both B & C

A 40 year old woman report with the complaint of burning sensation in the mouth. Clinical examination reveals lesions consisting of radiating white striations in a retiform arrangement affecting buccal mucosa, tongue, lips & gingiva bilaterally. An incisional biopsy is suggestive of lichen planus. The following are different clinical forms of lichen planus except:

A. Atrophic lichen planus
B. Hypertrophic lichen planus
C. bullous lichen planus
D. Verrucous lichen planus

Intraepithelial vacuolation with formation of vesicle or bulla intraepithelially above the basal layer is characteristically seen in:___ ___?

A. Candida albicans
B. Bullous pemphigoid
C. Pemphigus
D. Lichen planus

Lichenoid reactions are mainly due to:___ ___?

A. intake of certain drugs
B. betel nut chewing
C. cigarette smoking
D. intake of alcohol

L.E Cell phenomenon in peripheral blood is seen in:____ __?

A. Rheumatic heart disease
B. Infective endocarditis
C. Ischemic heart disease
D. Systemic Lupus Erythematosus

Which of the following is inherited as a autosomal dominant triat?

A. Lichen planus
B. Bullous pemphigoid
C. Pemphigus yulgaris
D. White sponge nevus

Histopathological study of lichen planus shows:___________?

A. Mixed cellular inflammatory infiltrate
B. Presence of T-lymphocytes predominantly
C. Antiepithelial antibodies
D. Scattered infiltrate with ill-defined lower border

Pemphigus is characterized by:__ __?

A. Acanthosis
B. Acantholysis
C. Hyperorthokeratosis
D. Hyperparakeratosis

In ectodermal dysplasia all of the following structures are affected except:___ ____?

A. Hair
B. Nails
C. Teeth
D. Salivary glands

Scleroderma involves:___ __?

A. Tightening of oral mucosa and periodontal involvement
B. Multiple palmar keratosis
C. Raynaud’s phenomenon
D. All of the above

Erythema multiformae is:____ __?

A. An acute self limiting disease, of skin and oral mucous membrane
B. painless vesicular self limiting disease
C. A viral disease
D. Bacterial infection

Darier’s disease is associated with:__ ____?

A. Pernicious anaemia
B. Rickets with involvement of teeth and bones
C. Vitamin A deficiency and involvement of oral epithelium and skin
D. Diffuse tender ulceration on the palate predominantly

Koebner’s phenomenon is seen with_______?

A. Erythema multiforme
B. Pemphigoid
C. Psoriasis
D. Impetigo

Tzancks smear test is used in the diagnosis of______?

A. pemphigus
B. ANUG
C. Apthous disease
D. Lichen planus

Unusual extensibility of the tongue is a characteristic feature of:__ ____?

A. Epidermolysis bullose
B. Syphilis
C. Darier-White disease
D. Ehlers-Danlos syndrome

Which of the following is not a type of lichen planus?

A. Atrophic
B. Hypertrophic
C. Verrucous
D. Erosive

A fluid filled elevated lesion of skin is called______?

A. Bulla
B. Macule
C. Papule
D. Nodule

Grinspan syndrome is associated with:___ __?

A. Hypertension, diabetes, lichen planus
B. Oral, ocular, genital lesions
C. Hypertension with oral lesions
D. lemphigus, CHF, diabetes

In lichen planus the basal cells which are shrunken with an eosinophilic cytoplasm and with a pyknotic and fragmented nuclei are called______?

A. Tzanck cells
B. Civatte bodies
C. Donovan bodies
D. Rushton bodies

MONRO’s abscess are seen in:___ ___?

A. Pemphigus
B. Lichen planus
C. Leukoplakia
D. Psoriasis

Nikolsky’s sign in positive in:___ ____?

A. bullous pemphigus
B. eipdermolysis bullosa
C. herpes simplex
D. erythema multiforme

Which of the following is absent in Crest syndrome?

A. calcinosis cutis
B. Raynaud’s phenomenon
C. Telagietasis
D. Endocrine disorders

Which of the following diseases of the skin is the most likely to be associated with partial anodontia?

A. erythema multiformae
B. hereditary actodermal dysplasia
C. Keratosis follicularils
D. lichen planus

Antinuclear antibodies are seen in:_ ____?

A. SLE
B. Systemic sclerosis
C. Morphea
D. All of the above

Which of the following is an oral manifestation of lichen planus?

A. Dentinogenesis imperfecta
B. Fordyce spots
C. White, chalky enamel surface
D. White radiating lines on the buccal mucosa

A twenty-one-year-old woman complains that regular, gentle brushing of her teeth is painful besides causing profuse bleeding. Oral examination reveals the loss of epithelium from the attached gingival of both arches. Which of the following dermatological problems is this patient most likely to have?

A. Benign mucous membrane pemphigiod
B. Chronic discoid lupus erythematosus
C. Pemphigus
D. Psoriasis

A flat, cricumscribed discolouration of skin or mucosa that may vary in size and shape is referred to as:___ ___?

A. Epulits
B. Macule
C. Nodule
D. papule

True about caries, all except:

A. Infectious and transmissible
B. Not due to microorganisms
C. Can develop in the absence of sucrose
D. Microorganisms play the most essential role

Plaque microflora can-split carbohydrates. What does it means?

A. Sacchrolytic
B. Saprophytic
C. Virulant
D. Avirulant

Which of the following represents soluble polysaccharide found in dental plaque and is formed from the fructose moiety of sucrose?

A. Levan
B. Dextran
C. Amlyopecting
D. Hyaluronic acid

Progression of dental caries caries on pit and fissure occurs from:

A. Apex of the pit and fissure
B. Wide and of the pit and fissure
C. Lateral surface of the pit and fissure
D. Bottom of the pit and fissure

Which of the following is cariostatic?

A. Selenium
B. Magnesium
C. Cadmium
D. Molybdenum

Liquefaction foci of Miller is a histopathological observation in:

A. Cemental caries
B. Early enamel caries
C. Advanced enamel caries
D. Advanced dentinal caries

Streptococcus mutans produces an adhesive polymer from sucrose, known as:____ ___?

A. Levans
B. Lectins
C. Glucans
D. Polyfructans

The gelatinous deposit adherent on the tooth surface is called as:__ ____?

A. Materia alba
B. Plaque
C. Calculus
D. All of the above

Most demineralised zone in enamel caries:____ ___?

A. Translucent zone
B. Body of lession
C. Dark zone
D. Surface zone

Most used selective medium for streptococcus mutans is:___ ____?

A. Mac conkey medium
B. Mitus salivarius bacitracin agar
C. Nutrient agar
D. Tellurite medium

The cells most frequently found in a granuloma are:__ ___?

A. Mast cells
B. Giant cells
C. Lymphocytes
D. Neutrophilis

Tiny linear or arc-shaped bodied, amorphous, brittle and eosinophilic in reaction found in association with some odontogenic cysts, are called:__ ____?

A. Civattle bodies
B. Russell bodies
C. Guarneri bodies
D. rushton bodies

Low grade infection which leads to localized periosteal reaction is:__ _____?

A. Garre’s osteomyelitis
B. Acute osteomyelitis
C. Condensing osteitis
D. Local alveolar osteitis

Cyst arising from rests of malassez is:__ ____?

A. Dental cyst
B. Dentigerous cyst
C. Radicular cyst
D. Karato cyst

The tooth most commonly involved in chronic focal sclerosing osteomyelitis is:_ ____?

A. Maxillary second molar
B. Maxillary third molar
C. Maxillary first molar
D. Mandibular first molar

Three stages in progression of acute odontogenic infection are:__ ____?

A. Periapical osteitis, cellulitis, abscess
B. Abscess, cellulitis, osteitis, Periapical
C. cellulitis, Abscess, Periapical, osteitis
D. Periapical osteitis, abscess, cellulitis,

The fascial spaces involved in ludwig’s angina are_______?

A. Unilateral – submandibular & sublingual spaces
B. Bilateral – submandibular & sublingual spaces
C. Unilateral – submandibular sublingual & submental spaces
D. Bilateral – submandibular sublingual & submental spaces

The chronic osteomyelitis of the jaw consists of:__ ____?

A. condensing osteitis
B. Sclerotic cemental mass
C. chronic diffuse sclerosing osteomyelitis
D. All of the above

Which of the following is more prone to osteomyelitis:__ __?

A. Maxilla
B. zygoma
C. palatine bone
D. mandible

Chronic periostitis in children is known as_____?

A. Cherubism
B. Garre’s osteomyelitis
C. Histiocytosis X
D. Tuberculous osteomyelitis

A diffuse spreading inflammatory lesion is due to bacterial enzyme

A. Coagulase
B. Hyaluronidase
C. Peroxidase
D. Bradykinin

Which of the following differentiates between condensing osteitis and benign cementoblastoma?

A. Condensing osteitis is associated with vital teeth where as cementoblastoma is associated with non-vital teeth
B. In condensing osteitis radiopacity is attached to tooth where as in cementoblastoma it is not
C. Cementoblastoma is associated with vital tooth where as condensing osteitis is associated with non-vital tooth
D. In cementoblastoma radiopacity is attached to tooth where as in condensing osteitis it is not

Constant feature associated with a radicular cyst

A. An impacted tooth
B. A missing both
C. A non-vital tooth
D. An anomalous tooth

The caries of enamel surface leads to accentuation of:__ ____?

A. Incremental lines of retzius
B. Perikymata
C. Imbrication lines of pickerill
D. Wickham’s striae

Odontogenic epithelium responsible for the formation of dental cyst is:___ __?

A. Cell rests of seirre
B. Enamel organ
C. Reduced enamel epithelium
D. Cell rests of malassez

Which is not true of Ludwig’s angina?

A. Usually arises from an infected molar
B. involves submandibular space
C. May need emergency tracheostomy
D. None of the above

Reversible pulpitis change to irreversible pulpitis primarily because of:___ __?

A. Vacular strangulation
B. Reduced host resistance
C. Invasion of microorganisms
D. An increase in microbial virulence

Pain due to acute irreversible pulpitis is:__ ___?

A. Spontaneous
B. Sharp- shock like
C. Lasting for short time
D. Continuous

A person experiences throbbing pain at night. It is due to:__ ___?

A. Acute Pulpal degeneration
B. Acute periodontal abscess
C. Chronic pulpitis
D. Cellulitis

An asymptomatic tooth has deep has deep caries on occlusal surface. Radiograph shows radiopaque mass at apex of the tooth: this mass is most likely to be:___ ____?

A. Cementoma
B. Condensing Osteitis
C. Chronic apical periodontitis
D. Acute apical periodontitis

Acute osteomyelitis is most frequently caused by which of the following microorganisms?

A. Gonococcus
B. Enterococcus
C. Streptococcus
D. Staphylococcus

Periapical cyst is usually preceded by:___ ____?

A. Periapical granuloma
B. Periodontal abscess
C. Periapical abscess
D. All of the above

How to prepare tests? See Below: 

1.     How to prepare Verbal Intelligence tests? Click Here
2.     How to prepare Non Verbal Intelligence tests? Click Here
3.     How to prepare Medical tests? Click Here

Oral Pathology & Medicine MCQs

Chronic hyperplastic pulpitis is:__ __?

A. Necrotizing
B. Suppurative lesion
C. proliferation of a chronically inflamed pulp
D. Also called as phoenix abscess

A tooth with a 3 month history of pain, which was worse when hot liquid were in mouth. After extraction, the tooth was split open. The pulp chamber was completely filled with pus. A few remnants of pulp tissue were found in apical end. The condition is:___ ____?

A. Acute partial pulpits
B. Acute total pulpits
C. Suppurative pulpitis
D. Strangulation of pulp

Focal sclerosing osteomyelitis is:___ __?

A. Due to excessive periosteal bone formation
B. An extremely painful condition
C. Due to low grade chronic infection
D. A common sequel following sequestrectomy

Osteomyelitis begins as an inflammation of:__________?

A. Cortical bone
B. Periosteum
C. Medullary bone
D. periosteum and inner cortex

The earliest radiographic sign of osteomyelitis is_____?

A. Solitary or multiple small radiolucent areas
B. Increased granular radioopacity
C. Blurring of trabecular outlines
D. Formation of sequestrum appearing as radiopaque patches

The most common organism involved in a periapical abscess is:__ __?

A. Strep. Pyogens
B. Strep. viridans
C. β hemolytic stretococci
D. Non hemolytic stretococci

The main causative agent of Ludwig’s angina is:__________?

A. Anaerobic streptococci
B. Aerobic streptococci
C. Staphylococci
D. Legionella infection

Which of the following periapical conditions is often associated with a vital pulp?

A. Apical cyst
B. Apical scar
C. Condensing osteitis
D. Chronic apical periodontitis

An acute apical abscess is usually a result of:__ __?

A. Periodontal pocket
B. Occlusal interference
C. Necrotic pulp
D. Chronic gingivitis

Abscess formation is particularly characteristic of infections with which of the following microorganisms:_____________?

A. Viruses
B. Rickettsiae
C. Streptococci
D. Staphylococci

Garre’s chronic non suppurative sclerosing osteomyelitis is characterized clinically by:__ ______?

A. Endosteal bone formation
B. Periosteal bone formation
C. Resorption of medullary bone
D. Resorption of cortical bone

The earliest response of pulpitis is:__ ___?

A. Cyst formation
B. Calcification
C. Hyalinization
D. Formation of dental granuloma

Dental cyst:___ _____?

A. Occurs from the reduced enamel epithelium
B. Replaces the tooth to which it it attached
C. Is frequently seen with a missing tooth on the X-ray
D. Cystic lining of stratified squamous epithelium

Most common cyst in oral region is:__________?

A. Medial cyst
B. Radicular cyst
C. Follicular cyst
D. Naso labial cyst

Phelogmon is a:__ ___?

A. Sexually transmitted disease
B. Type of cellulites
C. Type of osteomyelitis
D. Venereal disease

All of the following statements about the typical features of a periapical granuloma are true EXCEPT:

A. It consists of proliferating granulation tissue
B. It can form only if the periapical bone is resorbed
C. It shows evidence of local antibody production
D. It results from immunologically mediated tissue damage

The recurrence of pleomorphic adenoma is attributed to______?

A. Presence of an incomplete capsule
B. Mixed origin
C. Absence of capsule
D. Perineural spread

Sjorgen’s syndrome includes all except_____?

A. Xerostomia
B. Keratoconjunctivitis
C. Arthritis
D. Lymphoma

Which tumour does not occur in minor salivary gland ?

A. Pleomorphic adenoma
B. Adenocarcinoma
C. Mucoepidermoid carcinoma
D. Warthin’s tumour

Sialolith in the excretory duct, will result in______?

A. Chronic sialadenitis
B. Mucous retention cyst
C. Pleomorphic adenoma
D. rupture of the duct

The most common complication of mumps is______?

A. myocarditis
B. Orchitis
C. Uveitis
D. Conjunctivits

A cyst occurs under the tongue, caused by obstruction of a salivary gland. Such a cyst is called_______?

A. Mucocele
B. Ranula
C. Dermoid cyst
D. Dentigerous cyst

Sialography is used to detect anomaly of______?

A. Salivary duct only
B. Salivary gland
C. Salivary gland & duct
D. Salivary gland tumours

Which of the following is of salivary gland origin ?

A. Acinic cell carcinoma
B. Granular cell myoblastoma
C. Chondrosarcoma
D. All of the above

A condition of the mouth with increase the caries activity in the oral cavity is______?

A. Xerostomia
B. Malignancy
C. Hairy tongue
D. Watery saliva

Most common tumor of parotid gland is_____?

A. Plemorphic adenoma
B. Adenoid cystic carcinoma
C. Cylindroma
D. Epidermoud carcinoma

Bimanual palpation technique is carried out for_________?

A. Submandibular gland
B. Sublingual gland
C. Ranula
D. Cervical lymph nodes when they are enlarged due to inflammation

The most common salivary gland malignant neoplasm in bones_______?

A. Plemorphic adenoma
B. Adenoid cystic carcinoma
C. Muceopidermoid carcinoma
D. Adenolymphoma

Warthins tumor is______?

A. An adenolymphoma of the parotid gland
B. A pleomorphic adenoma of parotid gland
C. Carcinoma of the parotid gland
D. None of the above

The common site for narcotizing sialometaplasia_______?

A. cheeks
B. dorsum of tongue
C. palate
D. gingival

In the clinical evaluation, the most significant, finding of the parotid mass may be accompanying_____?

A. Rapid progressive painless enlargement
B. Nodular consistency
C. Supramental and preauricular lymphadenopathy
D. Facial paralysis

Salivary gland aplasia is seen in_______?

A. Hemifacial microstomia
B. LADD syndrome
C. Mandibulo-facial dysostosis (Treacher Collins)
D. All of the above

All of the following is the extraglandular manifestation of primary sjogren’s syndrome except_________?

A. Raynaud’s phenomena
B. Arthritis
C. Lymphadenopathy
D. Thrombocytopenia

“Xerostomia” is seen in all of the following EXCEPT in_______?

A. Anticholinergic drugs
B. Dehydration
C. Sjogren’s syndrome
D. Oral sepsis

Commonest salivary gland tumour in children_____?

A. Lymphoma
B. Pleomorphic adenoma
C. Adenoid cystic carcinoma
D. Mucoepidermoid carcinoma

Spindle cell carinoma is a variant of_______?

A. Pleomorphic Adenoma
B. Adenoid cystic carcinoma
C. Basal cell carcinoma
D. Squamous cell carcinoma

Commonest site for ectopic salivary gland tumour is________?

A. Tongue
B. Cheek
C. Palate
D. Neck

Chemical mumps’ is synonymous with________?

A. Epidemic parotitis
B. Iodine mumps
C. Nutritional mumps
D. Nonspecific mumps

Salivary duct calculi_________?

A. produce pain on eating
B. Are commonest in the parotid ducts
C. Are common cause of acute parotitis
D. Are associated with hypercaleaemic states

In Xerostomia the salivary pH is_____?

A. Unaffected
B. Low
C. High
D. Increased in morning & decrease in day

Which of the following statements about sjogren’s syndrome is incorrect_______?

A. MRI shows salt and pepper appearance
B. Lacrimal gland enlargement is common
C. Minor salivary gland biopsy is diagnostic test
D. Pilocarpine is the most useful and convenient drug in its treatment

Intra Oral pleomorphic adenoma usually occurs in________?

A. Buccal mucosa
B. Upper lip
C. Floor of the mouth
D. Palate

Sjogren’s syndrome affects_____?

A. Exocrine glands
B. Paracrine glands
C. Endocrine glands
D. Autocrine glands

The following are the high grade salivary gland malignancies, EXCEPT________?

A. Basal cell adenocarcinoma
B. Mucoepidermoid carcinoma
C. Adenoid Cystic carcinoma
D. Salivary duct carcinoma

Which of the following salivary gland tumors has highest rate of malignant transformation?

A. Patrotid
B. Submanidbular
C. Sublingual
D. Minor

The following are the high grade salivary gland malignancies, EXCEPT_____?

A. Basal cell adenocarcinoma
B. Mucoepidermoid carcinoma
C. Adenoid Cystic carcinoma
D. Salivary duct carcinoma

In which one of the following salivary gland tumors, the tumor is composed of ” intermediate cells” histologically?

A. Adenoid cystic carcinoma
B. Mucoepidermoid carcinoma
C. Pleomorphic adenoma
D. Warthin’s tumour

A salivary gland tumour, which histologically shows a double layer of epithelial cells based on a reactive lymphoid stroma is______?

A. Pleomorphic adenoma
B. Mucoepidermoid carcinoma
C. Acinic cell tumour
D. Warthin tumour

Chocolate-coloured fluid is seen in the cystic space of_________?

A. Dentigerous cyst
B. Odontogenic deratocyst
C. Unicystic ameloblastoma
D. Papillary cystadenoma lymphomatosum

Pleomorphic adenoma is_____?

A. a teratomatous tumour of the salivary gland
B. a neuroendocrine cell tumour
C. multiple ectopic tissue proliferating in the salivary gland
D. myoepithelial or ductal reserve cell origin

Sialoliths are stones found in the salivary duct and gland and are primarily composed of_______?

A. Hydroxyapatitie
B. Potassium cloride
C. Unknown compounds of phosphats
D. Calcium chloride

Break up time (BUT) test is done in______?

A. Sjogren’s syndrome
B. Multiple sclerosis
C. SLE
D. Myasthenia gravis

Commonest site for ectopic salivary gland tumour is_________?

A. Tongue
B. Cheek
C. Palate
D. Neck

Adeno lymphoma refers to_______?

A. Adeno carcinoma
B. Adeno cystic lymphoma
C. Warthin’s tumour
D. Pleomorphic adenoma

Mixed tumour of the salivary glands are_____?

A. Most common in submandibular gland
B. Usually malignant
C. Most common in parotid gland
D. Associated with calculi

Acinic cell carcinomas of the salivary gland arise most often in the_______?

A. Parotid salivary gland
B. Minor salivary glands
C. Submandibular salivary gland
D. Sublingual salivary gland

Xerostomia, enlargement of salivary and lacrimal glands is seen in________?

A. Sicca syndrome
B. Sjogren’s syndrome
C. Mickulicz’s disease
D. None of the above

Saliva is increased by______?

A. Cholinergic drugs
B. Anticholinergic drugs
C. Andrenergic drugs
D. None of the above

Tender submandibular swelling is mostly due to________?

A. Ludwig’s angina
B. Stone or Sialolithiasis
C. Enlarged lymph nodes
D. All of the above

Leafless fruit laden tree or cherry-blossom appearance on a sialogram indicates________?

A. Mucoepidermoid cell carcinoma
B. Acinar cell carcinoma
C. Sjogren’s syndrome
D. Pleomorphic adenoma

Parotid fatty change is sign of________?

A. Aging
B. Alcoholism
C. Malnutrition
D. None of the above

Adenoid cystic carcinoma is also known as_______?

A. Cylindroma
B. Pindborg tumor
C. Warthins tumor
D. Pleomorphic adenoma

Mucocele most commonly arise as a result of_______?

A. Rupture of a salivary duct
B. Partial or complete compression of the salivary acini
C. Inflammatory changes in the glandular interestitial tissue
D. Partial or complete obstruction of teh salivary duct by calculus

Acute non-suppurative sialdenitis is seen in_______?

A. Acute bacterial sialadenitis
B. Mumps
C. Chronic backerial sialadenitis
D. Necrotizing sialometaplasia

Reduction in flow of saliva is not generally seen in________?

A. Elderly diabetics
B. Patient undergoing radiation therapy
C. Patients suffering from parkinsonism
D. Patients on phenothiazine drugs

Which of the following parotid malignancy shows perineural spread________?

A. Pleomorphic adenoma
B. Adenoid cystic carcinoma
C. Warthin’s tumor
D. Ductal papilloma

A painful crater like 1.5 cm ulcer develops within one week on the hard palate mucosa of a 40 year old female The most likely diagnosis is_______?

A. Actinomycosis
B. Squamous cell carcinoma
C. Pleomorphic adenoma
D. Necrotizing sialometaplasia

Mikulicz’s disease is______?

A. An inflammatory disease
B. Neoplastic disease
C. An autoimmune disease
D. Viral infection

Pleomorphic adenoma arises from_______?

A. Myoepithelial cells
B. Aciner cells
C. Connective tissue
D. Stem cells

Which of the following statement is FALSE?

A. A salivary duct obstruction can cause a unilateral swelling in the floor of the mouth that is largest before a meal and smallest after a meal
B. The lesion termed a ranula is associated with the sublingual salivary gland
C. The sublingual salivary gland is the most common site of salivary gland neoplasia
D. A pleomorphic adenoma is the most common salivary gland neoplasm

Salivary gland stone most commonly involves____?

A. Submandibular gland
B. Parotid gland
C. Sub lingual glands
D. Lingual glands

Severe pain which arise after injury to or sectioning of a peripheral sensory nerve is called as________?

A. Temporal arteritis
B. Neuralgia
C. Neuritis
D. Causalgia

Patient suffering form Eagle’s syndrome complains of________?

A. burning sensations in mouth
B. excessive salivation
C. Glossodynia
D. Dysphagia

The latest drug of the choice in the management in trigeminal neuralgia is________?

A. valproic acid
B. carbamazepine
C. Diphen hydantoin
D. None of the above

Facial paralysis is tested by_______?

A. Whistling
B. chewing
C. Protruding the tongue
D. Swallowing

Which of the following structures are associated with bells palsy________?

A. sub mandibular gland
B. Seventh cranial nerve
C. Temporomandibular joint
D. Glosso pharyngeal nerve

If a patient with Raynaud’s disease puts his hand in cold water, the hand appears_________?

A. Red
B. Yellow
C. White
D. Blue

Geniculate neuralgia is caused in the nerve________?

A. VII
B. IX
C. X
D. II

All of the following are true about trigeminal neuralgia EXCEPT_______?

A. it is unilateral
B. it is of throbbing nature
C. it is triggered by touching cheeks, mucosa etc
D. occurs in bouts

Trotter’s syndrome involves________?

A. Pharynx
B. Oropharynx
C. Larynx
D. Nasopharynx

“Fothergill’s disease” is one of the synonyms of_______?

A. Sarcoidosis
B. Multiple sclerosis
C. Trigeminal neuralgia
D. Lupus erythematosis

Lesion of facial nerve at level of stylomastoid foramen leads to_______?

A. Loss of taste sensation from Ant. 2/3 of tongue
B. Paralysis of orbicularis oculi muscle
C. Loss of innervation to stapedius
D. Loss of lacrimal secretion

What is non characteristic of Eagle’s syndrome________?

A. Excessive lacrimation
B. pain during mandibular movement
C. Stabbing type pain orginate in the tonsillar regions
D. When the jaws are closed the pain subsided

Which of the following drugs is not effective in case of Trigeminal Neuralgia ?

A. Carbamazipine
B. acetaminophen
C. phenytoin sodium
D. Baclofen

Patient comes with pain pharyngeal region and is having carcinoma of nasopharynx. The diagnosis is ______?

A. Horner’s syndrome
B. Glossopharyngeal neuralgia
C. Trotter’s syndrome
D. Eagles syndrome

The characteristic alarm clock headache is a feature of_____?

A. Auriculotemporal Neuralgia
B. Trigeminal Neuralgia
C. Sphenopalatine Neuralgia
D. Glossopharyngeal Neuralgia

Anti-convulsants frequently used in management of trigeminal neuralgia are________?

A. Phenytoin
B. Gabapentin
C. Baclofen
D. All of the above

Which of the following Orofacial pain is not associated with vascular origin ?

A. Cluster headache
B. Giant cell arteritis
C. Anaesthesia dolorosa
D. Chronic paroxysmal hemicrania

An attack of cluster headache can be aborted by _____?

A. Morphine administration
B. Breathing oxygen
C. Aspirin administration
D. Sublingual nitroglycerine administration

Burning Mouth Syndrome describes pain associated with_____?

A. Oral lichen planus
B. Oral submucous fibrosis
C. Aphthous stomatitis
D. No detectable oral disease

In an acute attack of migraine, the during of choice_______?

A. Ergontamine tortrate
B. Methysergide
C. Propranolol
D. Caffeine

Mask-like appearance of face with narrowing of aperture and rigidity of the mucosa is characteristic of________?

A. Progressive systemic sclerosis
B. Tetanus
C. Multiple sclerosis
D. Osteomalacia

A patient shows inability to close the right corner of the mouth is most probably suffering form_______?

A. Myasthenia gravis
B. Bell’s palsy
C. TMJ dysfunction syndrome
D. Multiple sclerosis

Bell’s palsy is triggered by_______________?

A. Exposure to cold
B. Tooth extraction
C. Local and systemic infection
D. Any of the above

The following site is the common involvement in case of Myositis ossificans_______?

A. Massetor
B. Hyoglossus
C. Stylohyoid
D. Lateral pterygoid

Trigeminal neuralgia_______?

A. Does not disturb the patient during sleep
B. Can be treated with NSAID’s
C. Always bilateral in distribution
D. Is a hereditary condition

Carbamazepine has been utilized to successfully diminish attacks in trigeminal neuralgia. During this therapy which of the following is indicated ?

A. Clinical observation only
B. Clinical observation and complete blood and platelet counts prior to and at frequent intervals during therapy
C. No monitoring
D. Complete blood investigation only if adverse symptoms arise

Which syndrome consists of flushing, warmness and perspiration over the cheek and pinna of the ear on the side following the ingestion of highly seasoned food ?

A. Fanconi’s
B. Auriculotemporal
C. Horner’s
D. Cushin’s

Facial pain due to elongated styloid process is called_______?

A. Cowden syndrome
B. Tic doulourex
C. Eagle’s syndrome
D. Reiter’s syndrome

A neuralgia with trigger zones in the oropharynx and pain in the ear pharynx, nasopharynx, tonsils and posterior tongue is most likely_______?

A. Trigeminal neuralgia
B. Bell’s palsy
C. Glossopharyngeal neuralgia
D. Sphenopalatine neuralgia

Easy fatigability of muscles seen in_____?

A. Epilepsy
B. MPDS
C. Myasthenia gravis
D. Cerebral palsy

Frey’s syndrome results from surgery of the _______?

A. Submandibular salivary gland
B. Parotid gland
C. sublingual salivary gland
D. TMJ

Horner’s syndrome Does NOT include______?

A. Ptosis
B. Anhydrosis
C. Flushing
D. Mydriasis

Trigeminal heuralgia (tic doulourex) is characterized by________?

A. Paralysis of one side of the face
B. Uncontrollable twitching of muscles
C. Sharp, excruciating pain of short duration
D. Prolonged episodes of plain on one side of the face

TENS therapy is useful in ______?

A. MPDS
B. Trigeminal neuralgia
C. Facial palsy
D. Neurosis

Bell’s Palsy is characterized by________?

A. Bilateral involvement of the side of the face
B. Inability to whistle
C. No loss of muscular control
D. Closing of the eyes

Which of the following are characteristic feature of cherubism________?

A. Premature exfoliation of primary teeth
B. Hypoplastic defects
C. Progressive painless symmetric
D. A + C

Precocious puberty is most characteristic of which of the following ?

A. Jaffe’s syndrome
B. Monostotic fibrous dysplasia
C. Abright’s syndrome
D. Osteogenesi imperfecta

Serum alkaline phosphatase levels are increased in_______?

A. Osteorthritis
B. Dentinogenesis imperfecta
C. Paget’s disease
D. Rheumatoid arthitis

Which of the following are a triad of the sign and symptoms of osteogenesis imperfecta________?

A. blue sclera, sparse hair, anhydrosis
B. enlarged hand, feet, maxilla, mandible
C. Blue sclera, brittle bones opalescent dentin
D. blue sclera, arachnodactyly, brittle bones

Generalised thickening of cortical and cancellous bones is seen in_______?

A. Osteopetrsis
B. Pagets disease
C. Osteogenesis imperfecta
D. Infantile hyperostosis

Cotton-wool appearance is seen in_______?

A. Peget’s disease
B. Osteoclerosis
C. Pariapical cemential dysplasia
D. Ossifying fibroma

Generalised hyper cementesis is seen in_______?

A. Hypophosphatasia
B. Pagets disease
C. Fibrous dysplasia
D. Cherubism

Class III malocclussion is seen in all of the following except________?

A. pierre robinson syndrome
B. Cleft palate
C. Cleidocranial dysplasia
D. Craniofacial dysostosis

Delayed eruption of teeth occurs in______?

A. Craniofacial dysostosis
B. Hyperthyroidism
C. Cleidocranial dysostosis
D. Osteitis deformans

False about cherubism________?

A. Unilocular lesion
B. Bilateral
C. Presence of Giant cell
D. Delayed eruption of permanent teeth

The histopathology of osteopetrosis shows______?

A. Endosteal bone formation and lack of normal bone resorption
B. Periosteal bone formation and lack of normal bone resorption
C. Persence of extra collagen fibres and less calcification resulting in resistance of bones to fracture
D. Presence of numerous osteoclasts and a few osteoblasts

Multiple fractures are seen in_______?

A. Rickets
B. Osteogenesis imperfecta
C. Osteomyelitis
D. Osteoma

IN MPDS which muscle is most apt to exhibit tenderness________?

A. Temporalis
B. Buccinator
C. Masseter
D. Lateral pterygoid

Orange peel and Ground glass radiographic appearance is observed in case of________?

A. Pagets disease
B. Weing’s sarcoma
C. Osteosarcoma
D. Fibrous dysplasia

Delayed dentition with multiple supernumerary teeth is seen in________?

A. Hypoparathyroidism
B. Cleidocranial dysplasia
C. Pierre Robin Syndrome
D. Mongolism (Down’s Syndrome

Mosaic pattern of bone is seen in radiographic features of______?

A. fibrous dysplasia
B. paget’s disease
C. Osteopetrosis
D. Osteogenesis imperfecta

The most common complication following rheumatold arthritis of the TMJ is_____?

A. Ankylosis
B. Synovial chondromatosis
C. Subluxation
D. Osteorthritis

Which of the following statement is false in relation to myofacial pain dysfunction syndrome ?

A. Maily affects young females
B. is caused by muscle fatigue due to chronic oral habits are grinding and clenching
C. Treatment involves construction of occlusal guard and stress free emotional condition
D. The perioral musculature becomes hypotonic

A child with Down’s syndrome has Moon facies, retarded mentally and which of the facial characteristic_________?

A. Maxillary prognathism
B. Mandibular retrognathia
C. Mandibular prognathism
D. Maxillary hypoplasia

Ground glass appearance in bone is seen in__________?

A. Hyper parathyroidism
B. Fibrous dysplasia
C. Condensing osteitis
D. Osteopetrosis

A non-neoplastic hereditary bone lesion, histologically similar to central giant cell granuloma affects children and shows, a bilateral involvement of the jaws with eye to heaven appearance clinically is______?

A. Fibrous dysplasia
B. Cherubism
C. Craniofacial dysostosis
D. Chondro-ectodermal dysplasia

Treacher collins syndrome is______________?

A. Maxillofacial Dysostosis
B. Mandibulofacial Dysostosis
C. Maxillomandibulofacial Dysostosis
D. Condylar Dysostosis

Osteosclerosis of bone occurs due to______?

A. Decreased host resistance
B. Increase in the virulence of organisms causes infection
C. Increased host response
D. Occurs in immunocompromised patients

Venous malformation involving the leptomeninges of the cerebral cortex is salient feature of________?

A. Rendu-Osler-Weber disease
B. Maffuci’s syndrome
C. Angioosteohypertrophy syndrome
D. Sturge weber syndrome

Preauricular pain, grating sensation and partial trismus are the symptons of________?

A. TMJ fibrous ankylosis
B. TMJ bony ankylosis
C. TMJ pain dysfurction symdnome
D. Ear infection

Polydactyly, craniosynostosis, late closure of fontanelles is a feature of_______?

A. Apert’s syndrome
B. Crouzon’s syndrome
C. Pierre robin syndrome
D. Down syndrome

Osteosarcoma characteristically may develop in some causes of________?

A. Osteopetrosis
B. Osteogenesis imperfecta
C. Acromegaly
D. Osteitis deformans

Histopathologically reversal lines are seen in_______?

A. Cherubism
B. Fibrous dysplasia
C. Paget’s disease of the bone
D. Craniofacial dysplasia

Most common primary malignant bone tumor is______?

A. Osteosarcoma
B. Ewing sarcoma
C. Metastatic carcinoma
D. Multiple Myeloma

Which of the following is NOT a clinical feature of Pierre-Robin syndrome ?

A. Micrognathia
B. Retrognathia
C. Glossoptosis
D. Coloboma of lower eyelid

Eruption fails in this bone disease, as there is no bone resorption______?

A. Primary hyperparathyroidism
B. Phantom bone disease
C. Paget’s disease
D. Osteopetrosis

Mutation in GNAS 1 gene is associated with_______?

A. Fibrous dysplasia
B. Ossifying fibroma
C. Focal cementoosseous dysplasia
D. Periapical cementoosseous dysplasia

The granulomatous tissue that is responsible for destruction of articular surfaces of TMJ in rheumatoid arthritis is known as_____?

A. Pannus
B. Pulse granuloma
C. Baker’s cyst
D. Immune granuloma

A patient aged 50 years presented with a history of jaw expansion and enlargement of maxilla_______?

A. paget’s disease
B. Acromegaly
C. Fibrous dysplasia
D. Hyperparathyroidism

Paget’s disease of bone is a chronic disease of the______?

A. Prepubertal skeleton
B. Pubertal skeleton
C. Infantile skeleton
D. Adult skeleton

A 15 years old boy reports with a rapidly growing swelling of the mandible with intermittent pain patient history is that the swelling occurred after an episode of trauma Radiographs reveal formation of new subperiosteal bone producing Onion skin appearance The patient also has lip paresthesia elevated white blood cell count Based on the clinical and radiographic picture, one of the following condition could be considered in the provisional diagnosis_______?

A. Fracture of the jaw with cancellous bone formation
B. Chronic suppurative osteomyelitis
C. Burkitt’s Lymphoma
D. Ewing’s sarcoma

Transformation into osteosarcoma is seen with_______?

A. pagets disease and polyostotic fibrous dysplasia
B. pagets disease and osteopetrosis
C. Cherubism and Polyostotic fibrous dysplasia
D. Cherubism and pagets disease

In a 60 year old adult, which of the following diseases causes expansile maxillary lesions ?

A. Paget’s disease
B. Acromegaly
C. Fibrous Dysplasia
D. Rickets

A patient with fibrous dysplasia can be treated by______?

A. Surgical excision
B. Removal of adjacent teeth
C. Irradiation of the lesion
D. Conservative surgery

The most likely diagnosis in a 23 year old, mentally alert, male dwarf with disproportionate arm and leg to body growth, prominent forehead and retruded maxilla is________?

A. Cretinism
B. Pituitary dwarfism
C. Acromegaly
D. Achondroplasia

In Cleidorcranial dysostosis, sometimes the roots of the permanent teeth are_______?

A. Thin and long
B. thin and short
C. Thick and short
D. Fused

A patient 18 years of age with sebaceous cysts on the scalp and back of the neck an osteroma on the right mandible: Radiographs reveal multiple impacted supernumerary teeth in both jaws. These findings suggest_______?

A. Cleidocranial dysostosis
B. Ectodermal dysplasia
C. Gardner’s syndrome
D. Osteogenesis imperfecta

During a routine checkup a 70 year old male is found to have Serum Alkaline Phosphatase three time the upper limit of normal. Serum Calcium, Serum Phosphorous and liver function tests are normal. The most likely diagonsis is_______?

A. Primary hyperparathyroidism
B. Paget’s disease of the bone
C. Osteomalacia
D. Metastatic bone disease

Immature bony trabeculae are found in______?

A. Fibrous dysplasia
B. Paget’s disease
C. Rickets
D. Cleidocranial Dysplasia

Albera-Schonberg’s disease is_______?

A. steomyelitis
B. Osteopetrosis
C. condensing osteitis
D. Osteomalacia

Pain, muscle tenderness, clicking or popping noise in T.M joint and limitation of jaw motion are the four cardinal signs and symptoms of______?

A. Costen’s syndrome
B. Traumatic arthritis of T.M J
C. Ostearthritis
D. Myofacial pain dysfunction syndrome

A 10-years-old child presents with anemia and recurrent fractures. The X-ray shows diffuse hyper density of bone The diagnosis is most likely to be______?

A. Osteogenesis imperfecta
B. Osteopetrosis
C. Osteochondroma
D. Hyperparathyroidism

Corticosteroids are useful in treatment of TMJ arthritis because they have_______?

A. An analgesic effects
B. An anti-inflammatory effect
C. Inhibitory effect on synovial membrane
D. Analgesic and anti-inflammatory effect

A patient with multiple impacted supernumerary teeth and can bring his shoulders together is suffering from_____?

A. Klinefelter’s syndrome
B. Trisomy 21
C. Down’s syndrome
D. Cleidocranial dysostosis

Normal serum, Ca nd alkaline PO4 are in________________?

A. Cherubism
B. Hypothyroidism
C. Hyperparathyroidism
D. Paget’s disease

Amber coloured tooth traslucency, blue sclerae and bone fragility and a history of pervious bone fractures are characteristic findings in______?

A. Osteoporosis
B. Osteogenesis imperfecta
C. Osteitis deformans
D. Osteitis fibrosa cystic

A patient complains of loss of visual acuity, deafness and enlargement of maxilla_______?

A. Paget’s disease
B. Osteomalacia
C. Fibrous dysplaisa
D. Osteogenesis imperfecta

The treatment for a child with cherubism is______?

A. surgical excision
B. cosmetic surgery after puberty
C. radiation
D. Enbloc dissection

Must see below:

1.     Best general knowledge questionsClick Here
2.     Top questions of InterviewClick Here
3.     Frequently asked Questions of InterviewClick Here
4.     Past Questions of Initial TestsClick Here
5.     Best Tips to pass InterviewClick Here

MCQs in Oral Pathology and Medicine with Answers

Biochemical abnormality associated with osteogenesis imperfecta is increase in______?

A. Alkaline phosphatase
B. Acid phosphatase
C. Bicarbonate ion
D. Phosphorylase enzyme

Of the following which is most common disorder causing pain about the masticatory apparatus including the TMJ________?

A. Traumatic arthritis
B. Trigeminal neuralgia
C. Myofacial pain dysfunction syndrome
D. Degenerative arthritis

Complication of Rheumatoid arthritis of condyle is______?

A. Fibrous ankylosis
B. Subluxation
C. Dislocation
D. None of the above

Alkaline phosphatase increases in______?

A. Pagets disease
B. Osteopetrosis
C. Cherubism
D. Fibrous dysplasia
E. A and D

The most common cause of TMJ ankylosis is_______?

A. Trauma
B. Osteoarthritis
C. Childhood illness
D. Rheumatoid arthritis

Down’s syndrome is associated with all except_______?

A. Retrognathia
B. Periodontal disease
C. Premature loss off deciduous teeth
D. Delayed eruption of deciduous teeth

The primary causative factor for myofacial pain dysfunction syndrome of the TMJ is_______?

A. Infratemporal space infection
B. Auriculotemporal neuritis
C. Muscular overextension and over contraction
D. Otitis media

A 3-year old patient reports of painless progressive bilateral facial swellings. The tentative diagnosis is_______?

A. Cherubism
B. Monostotic fibrous dysplasia
C. Ployostotic fibrous dysplasia
D. Central giant cell granuloma

Clavicle is absent is_______?

A. Osteogenesis imperfecta
B. Cleidocranial dysostosis
C. Fibrous dysplasia
D. Osteopetrosis

In a middle – aged man the radiograph shows cotton-wool appearance and blood investigation reveal an elevated alkaline phosphatase levels. The tentative diagnosis is______?

A. Paget’s disease
B. Cherubism
C. Fibrous dysplasia
D. Osteogenesis imperfecta

Which of the following has the potential of undergoing Spontaneous malignant transformation ______?

A. Osteomalacia
B. Albright’s syndrome
C. Paget’s disease of bone
D. Osteogenesis imperfecta

Which of the following diseases of the bone characteristically exhibits (in contrast to the other three conditions) a single lesion in a single bone ?

A. Central giant cell granuloma
B. Osteopetrosis
C. Paget’s disease of the bone
D. Polyostotic fibrous dysplasia

Osteogenesis imperfecta______?

A. In a se*-linked disorder of bones that develop that develop in cartilage
B. manifests with blue sclera which are pathognomonic of this disease
C. May be associated with deafness
D. Has associations with gametogenesis imperfecta

Static bone cyst is a cyst developing from_____?

A. Infection of Salivary gland in the mandible
B. Tissue of the odontogenic apparatus
C. Tissue of the oral mucosa
D. None of the above

A 6 year old patient with extra cusp on maxillary central incisor is associated with all except_______?

A. Mohr’s syndrome
B. Sturge-Weber Syndrome
C. Rubinstein Taybi Syndrome
D. Proteus Syndrome

A 4 year old child with one tooth less than the normal dentition shows large tooth with two crowns and two root canals, the anamoly is________?

A. Dilaceration
B. Fusion
C. Gemination
D. Concrescence

The cyst which is found within the bone at the junction of teh globular process, the lateral nasal process & maxillary process is______?

  1. Naso-alveolar cyst
    B. Globulomaxillary cyst
    C. Naso palatine cyst
    D. Mid palatine cyst

The cyst located at the junction of medial nasal process, lateral nasal process and maxillary process is_________?

  1. Globulomaxillary cyst
    B. Median palatine cyst
    C. Nasopalatine cyst
    D. Nasoalveolar cyst

The torus mandibularis most commonly seen in the region of_______?

A. Incisor region
B. Canine region
C. Molar region
D. Premolar region

Dentinal union of two embryologically developing teeth is referred as_______?

A. Gemination
B. Twinning
C. Concrescence
D. Fusion

Which one of the following is the cause of dilacerations ?

A. Trauma to the tooth germ during root development
B. Abnormal displacement of tooth germ during root development
C. Abnormal proliferation of enamel epithelium during tooth development
D. Abnormal displacement of ameloblasts during tooth formation

Taurodontism is usually seen in_______?

A. Mesiodens
B. Incisor with talon/cusp
C. Mandibular first molar
D. Maxillary premolars

Large pulp chambers are characteristic of all the following conditions except______?

A. Shell teeth
B. Tauradontisam
C. Dentin Dysplasia
D. Dentinogenesis imperfecta

The inheritance pattern of dentinogensis imperfecta is______?

A. Homozygous
B. Autosomal dominant
C. Recessive
D. X-linked recessive

Which of the following teeth is most likely to be congenitally missing ?

A. Maxi. central incisor
B. Mandibular canine
C. Mand. second premolar
D. Maxillary first premolar

In Treacher collin’s syndrome there is________?

A. Upward sloping of the palpebral fissure
B. Poorly developed or absence of malar bones
C. Progenia and mandibular prognathism
D. No loss of hearing

A bony hard asymptomatic swelling found on the midline of the hard palate that appears radiopaque on a radiograph is most likely a_______?

A. Odontoma
B. Myxoma
C. Boney cyst
D. Torus palatinus

Talon’s cusp is characteristic of which syndrome_______?

A. Edward’s syndrome
B. Klinefelter’s syndrome
C. Rubinstein Taybi syndrome
D. Down’s syndrome

Which of the following is unlikely to cause enamel hypoplasia ?

A. Rickets
B. Fluoride
C. Congenital syphilis
D. Cleidocranial dysostosis

Shell teeth are more common in this variant of dentinogenesis imperfecta______?

A. Type I
B. Type III
C. Type II
D. Type IIII and I

The 2nd common most supernumerary teeth is______?

A. Mesiodens
B. Distal to 3rd molar in maxilla
C. Distal to mandibular 3rd molar
D. Para molars

Estein Pearls are_______?

A. Gingival cyst of newborn
B. Gingival cyst of adult
C. Enamel pearls
D. Epithelial rests

A 15-year old boy shows an inverted pear shaped radiolucency between the upper central incisors. The teeth are normal in all aspects The most likely diagnosis is______?

A. Globulomaxillary cyst
B. Nasopalatine cyst
C. Aneurysmal cyst
D. Dentigerous cyst

In children the most frequently missing permanent teeth are________?

A. First premolars
B. Second Premolars
C. Max. lateral incisors
D. Mandibular lateral incisors

Estesin pearls are cysts that arise from________?

A. Squamous tissue of the mucosa
B. Connective tissue of the mucosa
C. Rests of malassez
D. Dental lamina

Median rhomboid glossits is associated is associated with______?

A. Oral cancer
B. fungal infection
C. Leukoplakia
D. Burning sensation of tongue

A 4-year old child has a normal complemented of primary teeth but they are gray and exhibit extensive occlusal and incisal wear. Radiographic examination indicates extensive deposits of secondary dentin in these teeth. Most likely this condition is______?

A. Neonatal hypoplasia
B. Amelogenesis imperfecta
C. Cleidocranial dysplasia
D. Dentinogenesis imperfecta

Geographical tongue, all are true except_______?

A. Lesions may be Bilaterally symmetrical on tongue
B. No treatment is required
C. Is a precancerous condition
D. May be related to emotional stress

Anodontia affects the growth of_____?

A. Maxilla
B. Mandible
C. Alveolar bone
D. Cranium

Which of the following is Fissural cyst is______?

A. Dentigerous cyst
B. Primordial cyst
C. Nasopalatine cyst
D. Radicular cyst

Taurodontism is characterized by______?

A. Hypercementosis
B. Elongated wide pulp canals and short roots
C. Obliterated pulp chambers with secondary dentine deposition
D. Rootless teeth with thin shell of enamel

Naso Labial cyst is thought to arise from______?

A. Remnants of cell rests of serrae
B. Remnants of cell rests of mallasez
C. Remnants of cell rests of embryonic lacrimalduct
D. Maxillary sinus lining epithelium

Hairy tongue is characterized by_______?

A. Hypertrophy of fungiform papillae
B. Hypertrophy of foliate papillae
C. Hypertrophy of filiform papillae
D. Hypertrophy of circumvallate papillae

Multiple osteomas, multiple polyposis, supernumerary teeth are found in _______ syndrome?

A. Reiter’s
B. Peutz – jeghers
C. Gardner’s
D. Behcet’s

Dental anomaly of teeth associated with defective bone formation is seen in________?

A. amelogenesis imperfecta
B. dentinogenesis imperfecta
C. Odontodysplasia
D. Osteitis deformans

Turners tooth is seen in_______?

A. enamel hypoplasia due to hypocalcemia
B. enamel hypoplasia due to birth injuries
C. enamel hypoplasia due to congenital syphilis
D. enamel hypoplasia due to local infection or local trauma

Bohn’s nodules are_______?

A. Cystic swellings in neonates
B. Cysts associated with soft palate
C. Cysts of gingiva in growing children
D. Warts on the tongue

Dentinogenesis imperfecta is________?

A. Autosomal dominant
B. Autosomal recessive
C. Se* linked recessive
D. Not a inheritable trait

Gardner syndrome does not include______?

A. Osteomas
B. Epidermoid cysts
C. Osteosarcoma
D. Impacted permanent teeth

Hypoplastic defects in permanent central and lateral incisors are likely to result due to severe illness or other factors during_______?

A. First nine month of life
B. First two years of life
C. First month of life
D. Two or three years of life

Fordyce’s spots are_______?

A. Fat tissue embedded in buccal mucosa
B. Red spots
C. Present on the cheek mucosa lateral to angle of the mouth
D. All of the above

The most common supernumerary tooth form is_______?

A. Tuberculated
B. Conical
C. Screw shaped
D. Incisor shaped

Absence of pulp chambers is seen in______?

A. Craniofacial dysostosis
B. Dentinogenesis imperfecta
C. Amelogenesis imperfecta
D. None of the above

Which of the following lesions does not give blood on aspiration______?

A. Heamagioma
B. Static bone cyst
C. Central giant cell granuloma
D. Aneursycimal bone cyst

A deep pit lined by enamel seen in the lingual surface of Maxillary lateral incipor is most likely to be_________?

A. Dens in dente
B. Enamel Hypoplasia
C. Talon’s cusp
D. Enamel Pearl

True generalized microdontia is characterized by_______?

A. Small teeth with large jaws
B. Small teeth with small jaws
C. Smaller teeth than the normal
D. Large teeth with small jaws

A permanent tooth with a local hypoplastic deformity in a crown is called_______?

A. Turner’s tooth
B. Taurodontism
C. Enameloma
D. Ghost teeth

Developing Ameloblasts are effected by the ingestion of high fluoride content water resulting in________?

A. Turners tooth
B. Hutchinson’s teeth
C. Mottled enamel
D. Moon’s molars

Which of the following is a soft tissue cyst which do not produce any radiographic changes______?

A. Nasolabial cyst
B. Nasopalatine cyst
C. Mid alveolar cyst
D. Palatine cyst

Which of the following features are of naso alveolar cyst______?

A. An avoid shaped radiolucency above the lateral incisor and canine teeth
B. Erosion of base above lateral incisor and canine teeth
C. An inverted funnel shaped radiolucent lesion above the roots of lateral incisor and canine teeth
D. A pear shaped radiolucent lesion between roots of lateralincisor and canine teeth

The term dilaceration refers to______?

A. A deformity of a tooth consisting of a sharp bend bend in the root
B. Abrasions on two surfaces of single tooth
C. A root or tooth that is split into two
D. A tooth that is fractured at two or more places

A girl suffering from browing of teeth, wearing of enamel but not cavitory, OPG shows obliteration of pulp with narrowing of canal & deposition of secondary dentin, she gave the history that out of her 4 brothers 2 are suffering from the same disease, She is suffering from_______________?

A. melogenesis imperfecta
B. Dentinogenesis imperfecta
C. Flurosis
D. Odontodysplasia

Failure of descent of thyroid analage can be seen in the tongue______?

A. In anterior 2/3 of dorsal aspect
B. In posterior 1/3 of dorsal aspect
C. Near the base of tongue close to foramen caecum
D. In anterior 2/3 of inferior surface

Geographic tongue is of red colour because of______?

A. Infiltration of eosinophils
B. Infiltration of neutrophils
C. Both of the above
D. None of the above

Developmental enamel defects are most commonly seen in________?

A. Primary incisors
B. Primary 2nd molar
C. Permanent incisors
D. Permanent 1st molar

Fordyce granules mostly occurs in all of the following area EXCEPT_______?

A. Vermilion of the lip
B. Posterior tonsillar pillar
C. Alveolar ridge
D. Palate

A 7-year old child having yellowish discoloured spot of maxillary central incisor. His mother presents a history of injury to deciduous tooth 3 years back with recurrent infection & swelling. The diagnosis is________?

A. urners hypoplasia
B. Generalised dental fluorosis
C. Syphillitic hypoplasia
D. Rickets hypomineralisation

Amelogenesis imperfecta is a disorder of_______?

A. Ectoderm
B. Mesoderm
C. Endoderm
D. Ecto and Mesoderm

Hypodontia is mainly found in all except______?

A. Papillion levefre syndrome
B. Osteopetrosis
C. Cleidocranialdysostosis
D. Ectodermal dysplasia

A child has marked difference of crown, root size, eruption pattern of left & right side of jaw. The condition is_______?

A. Crouzan syndrome
B. Hemifacial hypertrophy
C. Cherubism
D. Achondroplasia

Marked reduction in amount of dentin, widening of predentin layer, presence of large area of interglobular dentin and irregular pattern of dentin is seen in_______?

A. Hypocalcified dentin
B. Odonto dysplasia
C. Dentin dysplasia
D. Dentinogenesis imperfecta

Brittle bone syndrome is caused by_______?

A. Improper synthesis of procollagen
B. Polymerisation of collagen
C. Increase osteoclastic activity
D. Increase fibroblastic activity

Based on the degree to which apical displacement of the pulpal floor is seen, Cynodont is the name given to______?

A. Hypotaurodont
B. Mesotaurodont
C. Hypertaurodont
D. Normal tooth

According to Veau’s classification of cleft lip & palate, isolated cleft palate falls under______?

A. Group I
B. Group II
C. Group III
D. Group IV

Premaxilla-premaxillary cyst_______?

A. Nasoalveolar cyst
B. Nasopalatine cyst
C. Incisive canal
D. Globulomaxillary cyst

Cysts associated with vital teeth are_______?

A. Dentigerous cyst, Globulomaxillary cyst, lateral periodontal cyst, OKC
B. Dentigerous cyst, Globulomaxillary cyst, radicular cyst
C. Dentigerous cyst OKC, radicular cyst
D. ‘B’ & ‘C’

Most common site for melanotic neuroectodermal tumour of infancy is (MNTI)___ ____?

A. Maxila
B. Mandible
C. Ethmoid bone
D. Cervical spine

Taurodontism is seen in _______?

A. Klinefelter’s syndrome
B. Sturge weber syndrome
C. Down syndrome
D. Turner syndrome

Which of the following is most common development cyst_______?

A. Nasopalatine cyst
B. Naso-alveolar cyst
C. Globulomaxillary cyst
D. Median palatal cyst

The histological appearance of “lava following around boulders in dentin dysplasia suggests_______?

A. Attempt to repair the defective dentin
B. Abrupt arrest to dentin formation in crown
C. Abnormal dentin formation in a disorganized fashion
D. Cascades of dentin to form root

The most common congenital defect of the face and jaws is_______?

A. Macrostomia
B. Fetal alcohol syndrome
C. Cleft lip and palate
D. Ectodermal dysplasia

Submerged teeth are_______?

A. Ankylosed teeth
B. Unerupted teeth
C. Impacted teeth
D. Intruded teeth

In Hypodontia, the most commonly affected tooth is______?

A. Permanent third molar
B. Permanent second premolar
C. Permanent lateral incisor
D. Permanent Canine

PARULIS is an inflammatory enlargement seen in______?

A. End of sinus tract
B. Extraction Socket
C. Due to irritation from calculus / over hanging restoration
D. None of the above

Puetz-Jegher syndrome is characterised by______?

A. Deafness
B. Multiple supernumerary teeth
C. Multiple intestional polyps
D. scleroderma

Dense in dente is most commonly seen in______?

A. Paramolars
B. Paramolars
C. Lateral incisors
D. Maxillary canine

Heck’s disease is another name for______?

A. Focal epithelial hyperplasia
B. Fibromatosis gingiva
C. Oral melanotic macule
D. Hereditary intestinal polyposis syndrome

Which of the following dentitions shows the highest frequency of ocurrence of supernumerary teeth ?

A. Maxillary deciduous dentition
B. Maxillary permanent dentition
C. Mandibular deciduous dentition
D. Mandibular permanent dentition

Pierre Robin syndrome is associated with_______?

A. Micrognathia
B. Cleft of the lip and plate
C. Tetrology of fallot
D. Syndactally

Which of the following conditions is characterized by abnormally large pulp chambers ?

A. Amelogenesis imperfecta
B. Regional odontodysplasia
C. Dentinogenesis imperfecta
D. Dentinal dysplasia type I

Thistle-tube appearance of pulp chamber is a feature of_______?

A. Cornal dentin dysplasia
B. Regional odontodysplasia
C. Dentinogenesis imperfecta
D. Amelogenesis imperfecta

Mulberry molars are characteristic features of________?

A. Severe flurosis
B. Trauma at the time of birth
C. congenital syphilis
D. Due to chronic suppurative abscess in over lying gingival tissue

Complete obliteration of pulp is seen in all except_______?

A. Type I dentinogenesis imperfecta
B. Type II dentinogenesis imperfecta
C. Type III dentinogenesis imperfecta
D. Dentin dysplasia

Turner’s hypoplasia most commonly affects_______?

A. Deciduous maxillary anteriors
B. Deciduous mandibular anteriors
C. Permanent maxillary anteriors
D. Permanent manibular anteriors

Which of the following is not hereditary_______?

A. Amelogenesis imperfecta
B. Cleidocranial dysostosis
C. Regional odontodysplasia
D. Dentinogenesis imperfecta

Odontodysplasia is most common in_________?

A. Mandibular premolar
B. Mandibular canine
C. Mandibular third molar
D. Maxillary central incisor

Hutchinson’s incisiors are present in_________?

A. congenital syphilis
B. Tertiary syphilis
C. Secondary syphilis
D. Acquired syphilis

A dens in dente is usually caused by_______?

A. An abnormal proliferation of pulp tissue
B. Denticle formation within the pulp tissue
C. A deep invagination of the enamel organ during formation
D. A supernumerary tooth bud enclaved within a normal tooth

Most commonly submerged tooth is_______?

A. Mand. Primary 1st moral
B. Mand. Primary 2nd moral
C. Maxi. Primary 1st moral
D. Maxi. Primary 2nd moral

A developmental abnormality characterized by the presence of fewer than the usual number of teeth is______?

A. Anodontia
B. Oligodontia
C. Microdontia
D. Dens is dente

Which of the following dental sequel is likely in child with a history of generalized growth failure (failure to thrive) in the first 6 month of life_______?

A. Retrusive maxilla
B. Enamel hypoplasia
C. Retrusive Mandible
D. Dentinogenesis imperfecta

Delayed eruption of at least part of dentition is a recognized feature of all of the following except______?

A. Rickets
B. Congenital hyperthyroidism
C. Cleidocranial dysplasia
D. cherubism

Bifid tongue a congenital anomaly occurs due to non-fusion of_______?

A. Tuberculum impar and lateral lingual swellings
B. Hypobranhiral eminence and tuberculum impar
C. The two lateral lingual swellings
D. Some of the above

Mottled enamel is due to_______?

A. Vitamin A deficiency
B. Excess of fluoride
C. Vitamin D deficiency
D. Teratogens

Microdontia is most commonly seen affecting_______?

A. Max. lateral incisor
B. Mand. Second premolar
C. Mand. Central incisor
D. Mand. first premolar

A 4-year old child has less number of teeth and lateral incisors with bifurcated roots with two root canals is called________?

A. Dilaceration
B. Concrescence
C. Fusion
D. Gemination

Prolonged administrator of broad spectrum antibiotics results in the formation of_________?

A. Black hairy tongue
B. Median rhomboid glossitis
C. Geographic tongue
D. Fissured tongue

Fusion of teeth is more common in_______?

A. Primary dentition
B. Permanent dentition
C. Mixed dentition
D. None of the above

All are true about supernumerary tooth except______?

A. May have resemblance to normal teeth
B. Disto molars doesn’t resemble any other tooth
C. Mesiodens is the most common supernumerary tooth
D. More common in mandible

False about anodontia_______?

A. May involve both the deciduous and the permanent dentition
B. In false anodontia tooth doesn’t undergo full development
C. May involve a single tooth
D. In total anodontia all teeth are missing

Clinical evidence of dentinogenesis imperfecta is_________?

A. Defective enamel and dentine
B. Defective dentine and obliterated pulp chamber
C. Increased rate of caries
D. Oligodontia

Lingual tonsils arise_______?

A. As developmental anomalies
B. From carcinomatous transformation
C. As a result of hyperpalsia
D. Due to repeated trauma in the area

Ectopic sebaceous glands in the mouth are called_______?

A. Linea alba buccalis
B. Heck’s disease
C. Lingual verices
D. Fordyce spots

Most common missing tooth in the permanent dentition is______?

A. Maxillary canine
B. Maxillary first molar
C. Mandibular second premolar
D. Mandibular first molar

A patient notices a well demarcated area of depapillation on his tongue which has been there for as long as he can remember, The most probable diagnosis______?

A. Median rhomboid glossitis
B. Geographic tongue
C. Black hairy tongue
D. Moeller’s glossitis

Dentinogensis imperfecta differs from amelogenesis imperfecta in that, the former is_____?

A. A hereditary disturbance
B. The result of excessive fluoride ingestion
C. The result of faulty enamel matrix formation
D. Characterized by calcification of pulp chambers and the root canals of the teeth

Ghost teeth is seen in which of the following________?

A. Dens is dent
B. Regional odontodysplasia
C. Dentin dysplasia
D. None of the above

Gemination of teeth occur due to_______?

A. Division of a single tooth bud after calcification
B. Division of a single tooth bud before calcification
C. Fusion of two teeth before calcification
D. Fusion of two teeth after calcification

True ankyloglossia occurs as a result of_______?

A. Union between tongue and floor of mouth
B. Absence of lingual frenum
C. Lingual frenum attached to the tip of tongue
D. Short lingual frenum

Teeth that erupt with in 30 days of birth are called________?

A. Natal teeth
B. Neonatal teeth
C. Primary teeth
D. Prenatal teeth

The syndrom which consists of cleft palate micrognathia and glossoptosis is known as_______?

A. Marfan’s syndrome
B. Crouzon’s syndrome
C. Paget’s disease
D. PierreRobin syndrome

Facial edema, cheilitis granulomatosa and a fissured tongue characterize which of the following syndromes ?

A. Frey
B. Melkerson-Rosenthal
C. Teacher Collins
D. None of the above

Globulamaxillary cyst is_____?

A. Soft tissue cyst present often between maxillary Lateral incisor and cuspid teeth
B. Often present between incisor and cuspid teeth but is a bone cyst
C. A cyst present between the midline of the palate
D. A cyst present in the incisive canal

Peg-shaped incisors which taper towards the incisal edge are typically seen in all of the following conditions EXCEPT_______?

A. Congenital syphilis
B. Rickets
C. Anhidrotic ectodermal dysplasia
D. Supernumerary teeth

A union of the roots of adjacent teeth through the cementum is referred to as_______?

A. Concrescence
B. Fusion
C. Gemination
D. None of the above

Animals maintained in germ free environmental did not develop caries even when fed on a high carbohydrate diet is given by_______?

A. Gottlieb
B. Miller
C. Synder
D. Orland and Fizgerald

Initiation of caries by streptococcus mutans is by the production of_______?

A. Dextranase and soluble dextran
B. Insoluble dextan and glucosyl transferase
C. soluble dextan and glucosyl transferase
D. None of the above

Ammonia causes_______?

A. Increase in plaque formation
B. Increase in calculus formation
C. Decrease in plaque formation
D. Causes precipitation of salivary proteins

Pioneer bacteria in dental caries are in_______?

A. Enamel
B. Dentin
C. Pulp
D. Cementum

Chemico-parasitic theory of dental caries is proposed by____?

A. Miller
B. G.V black
C. Gottlieb
D. Schwartz

In a caries-free individual the saliva has_____?

A. low buffering capacity of acids
B. Medium buffering capacity for acids
C. High buffering capacity for acids
D. Independent of buffering capacity for acids

The lateral spread of dental caries is facilitated mostly by the_______?

A. Enamel spindles
B. Dentinoenamel junction
C. Enamel lamellae
D. Striae of Retzius

Caries, all are true except_______?

A. Lactobacillus is main acusative organism in plaque
B. Smooth surface caries occur due to streptococcus mutans
C. Pit and Fissure caries can be prevented by using pit and tissue sealants
D. Fluorides help in reducing caries incidence

Intitation of dental caries depends upon________?

A. Formation of large amount of acid
B. Availability of carbohydrate food
C. Viscosity of saliva
D. Localization of acid over tooth surface

Miller put forth the acidogenic theory of dental caries in the year_______?

A. 1890
B. 1920
C. 1924
D. 1980

Turbid dentin in carious tooth is all EXCEPT_______?

A. Zone of bacterial invasion
B. Zone which can not be remineralized
C. Zone in which collagen is irreversibly denatured
D. Zone that need not be removed before restoration

Least cariogenic among these______?

A. Raw starch
B. Cooked starch
C. Sucrose
D. Fructose

In the earliest stages of carious lesion. There is loss of______?

A. Enamel cuticle
B. Interprismatic substance
C. Organic matrix
D. Enamel lamellae

What is the PH at which initiation of caries begin ?

A. 4.3-4.5
B. 5.2-5.5
C. 4.9-5.1
D. 3.5-4.5

The dietary carbohydrate most likely involved in etiology of dental caries in man is_____?

A. Glucose
B. Sucrose
C. Dextran
D. Polysaccharide

The corrct order of microscopic zones of dentinal caries starting from the D.E junction is______?

A. Zone of sclerosis, decalcification zone, bacterial invasion
B. Bacterial invasion, decalcification zone, zone of sclerosis
C. Zone of sclerosis, backterial invasion, decalcification zone
D. Decalcification zone, zone of scierosis, backterial invasion

Dental caries is associated with______?

A. Streptococcus agalactiae
B. Streptococcus mutans
C. Streptococcus bovis
D. Streptococcus anginosus

Which of the following is cariogenic ?

A. Selenium
B. Vanadium
C. Strontium
D. Molybdenum

Starch is considered to be less cariogenic than monosaccharides and disaccharides because it________?

A. Does not diffuse through plaque
B. Is rapidly hydrolysed in the mouth
C. Enhances remineralisation
D. Raises the PH in the oral cavity

Organism involved in smooth surface caries is______?

1 Streptococcus mutans
2 Actinomyces viscosus
3 Lactobacillus
4 Campylobacter

A. only 1
B. only 2
C. 1 and 3
D. All of the above

The attachment of the Actinomyces species to the tooth surface is facilitated by_______?

A. Fimbriae
B. Cilia
C. Flagella
D. Pseudopodia

Which of the following factors in the stephan’s curve is related to the caries incidence and sugar intake______?

A. Physical form of sugar
B. Frequency of sugar intake
C. Ph of Plaque
D. Quantity of sugar intake

Cavity formation in a tooth, due to dental caries is due to______?

A. Destructive potential of streptococcus mutans
B. Destructive potential of lactobacillus acidophilus
C. Lateral spread of caries along DE junction and weakening of the outer covering enamel
D. Mastectomy force and unrelated to the extent of carious process

The most pronounced effect on the oral microflora of a reduction in rate of salivary flow is a______?

A. Significant increase in number of oral bacteria
B. Shift towards more acidogenic microflora
C. Significant decrease in number of oral bacteria
D. Shift towards more aerobic microflora

Which of the following organisms is found in deep carious lesions rather than in incipient lesions ?

A. Streptococci
B. Lactobacilli
C. Veillonella
D. Bateroides

Early invading bacteria in carious lesions are called_____?

A. Microcosm
B. Pioneer bacteria
C. Advancing bacteria
D. Anaerobic bacteria

The probable reasons for a high incidence of dental caries in the teenage population relates most directly to________?

A. Rapid growth
B. Frequency of sucrose intake
C. Negligence in visiting the dentist
D. Carelessness in oral hygiene habits

The enzyme glucosyl transferase secreted by Streptococcus mutans sythesizes glucans from_______?

A. Glucose
B. Fructose
C. Sucrose
D. Lactose

For a bacterium to be seriously considered in the etiology of dental caries, it must_______?

A. Exist regularly in the dental plaque
B. Produce extracellular amylopectins
C. Be lethal for gnotobiotic animals
D. Produce intracellular dextrans

Streptococcus mutans is considered to be principle etiologic agent of caries because it produces organic acids and it ______?

A. Forms a gelatinous matrix
B. Metabolizes substrate from saliva
C. Derives energy from enamel constituents
D. Lives symbiolically with lactobacillus

Which tooth in the permanent dentition is the most susceptible to dental caries ?

A. Maxillary 1st premolar
B. Maxillary 2nd molar
C. Mandibular 1st molar
D. Mandibular 2nd molar

Smooth surface caries is characterized by spread of caries in enamel and dentin as cones. These alignment in enamel and dentin is_______?

A. Base to base
B. Apex to base
C. Apex to apex
D. None of the above

Oral foci of miller’s are seen in_______?

A. Dental caries
B. Lichen planus
C. Herpes simplex
D. Syphilis

Check it for detail: 

1.     Join Pak Army as an Army OfficerClick Here
2.     Join Pakistan Navy as a Navy OfficerClick Here
3.     Join PAF as a PAF OfficerClick Here

Pakistan Army Short Service Regular Commission Tests of M-Cadet

Bacteria responsible for initiation of caries is _______?

A. Actinomyces
B. Strep. mutans
C. Lactobacillus
D. Strep. viridans

mutans is involved in dental caries initiation Other bacteria also involved is______?

A. S. Sarcinus
B. S. macae
C. S. sanguis
D. S. salivarius

Widely accepted theory of dental caries_______?

A. Proteolytic theory
B. Proteolytic chelation theory
C. Acidogenic theory
D. Autoimmune theory

The number of dentinal zones in dentinal caries is______?

A. One
B. Two
C. Four
D. Five

In a patient with reduced salivary flow the carious incidence is_______?

A. More than the patient with normal salivary flow
B. less than the patient with normal salivary flow
C. unaffected
D. None of the above

The extra – cellular polysaccharide sythesized by cariogenic streptococci in the presence of excess sucrose are best described as_____?

A. Mucopolysaccharide
B. Dextran-like glucan
C. Glycogen – like glucan
D. Amylopectin

Lactobacilli are numerous in caious lesions because they______?

A. Are the main causative agent
B. Can produce insoluble extracellular polysaccharides
C. Can attack to smooth enamel surfaces
D. Are secondary invaders

Tumor represented T2 N2 M0 is classified as______?

A. Stage I
B. Stage II
C. Stage III
D. Stage IV

Sturge Weber syndrome is characterized by all EXCEPT________?

A. Exophytic oral Hemangioma
B. Facial hematoma
C. Tramline calcification of dura on lateral cephalogram
D. Facial hemangioma

Keratocanthoma is found most commonly on ?

A. Lip
B. Gingiva
C. Tongue
D. Hard palate

Clear cells are seen in______?

A. Chondrosarcoma
B. Osteosarcoma
C. Fibrosarcoma
D. Ewing’s sarcoma

Neoplastic transformation in leucoplakia is seen most commonly in _____?

A. Buccal mucosa
B. Floor of mouth
C. Lateral border of tongue
D. Palate

Plasma cell tumour of bones with B-lymphocytic origin is ______?

A. Plasmacytoma
B. Multiple myeloma
C. Burkitt’s lymphoma
D. None of the above
E. Both A & B

The following lesion is a doubtful premalignant lesion for oral malignancy______?

A. Leukoplakia
B. Chronic hyperplastic candidiasis
C. Erythroplakia
D. Oral lichen planus

Definitive diagnosis of oral cancer is made by_______?

A. Complete radiographic survey
B. Biopsy
C. Exfoliative cytology
D. Pantograph

Diffuse erythema (pinpoint petechiae) and white patches in smoker’s palate occur due to________?

A. Complete obstruction to minor salivary gland orifices
B. Keratinization with partially occluded minor salivary glands
C. Diffuse parakeratinization of the palate
D. Excessive orthokeratinization of the palate

Histochemical demonstration of glycogen in the cells can help in the diagnosis of_______?

A. Malignant melanoma
B. Squamous cell carcinoma
C. Kaposi’s sarcoma
D. Ewing’s sarcoma

On biopsy report of CGCG on the basis of histologically & morphological similarities differential diagnosis is made between_____?

A. Fibrous dysplasia
B. Hyperparathyroidism
C. Osteitis deformans
D. Hyperthyroidism

Firbroma combined with glandular tissue is________?

A. Neurofibroma
B. Fibroadenoma
C. Fibrolipoma
D. Fibromyoma

Which of the following is false about ossifying fibroma ?

A. Affects people in 3-4 decade of life
B. Maxilla affected more than the mandible
C. Circumscribed radio opacity
D. Continuous growth

Treatment for verrucous carcinoma is_______?

A. Surgery + radiotherapy
B. Excision
C. Laser technique
D. Alpha (a) interferon

Which of the following is not a type of leukoplakia________?

A. Ulcerative
B. Bullous
C. Speckled
D. Homogenous

Benign tumor which shows metastasis_______?

A. Warthin’s tumor
B. Ameloblastoma
C. Keratocanthoma
D. Neurofibroma

Lateral skull view of palatal tori is used to know______?

A. Extent
B. Pneumatization
C. Bone pattern
D. Location

Presence of Epstein Barr virus in hairy leukoplakia can be demonstrated using following methods EXCEPT_______?

A. Tzanck smear
B. Polymerase chain reaction
C. In situ hybridization
D. Electron microscopy

Hairy laukoplakia is associated with all of the following EXCEPT_______?

A. Filiform to flat patch on lateral tongue
B. Bilateral appearance
C. Epstein Barr virus
D. AIDS

Oral ulceration resembling Apthae are encountered in______?

A. Gluten enteropathy
B. Chronic smokers
C. Excess of B-complex
D. Anti-malarial medication

Brachytherapy means _____?

A. Chemotherapy
B. Radiation administered interstially through catheters
C. Lasers
D. Radiation involving normal tissues

Commonest site of carcinoma of tongue_______?

A. Posterior one third
B. Ventral surface
C. Tip of tongue
D. Lateral margin

A patient with carinoma cheek has tumor of 2.5 cms located close to and involving the lower alveolus. A single mobile homolateral node measuring node measuring 6 cm is palpable. Based on these clinical findings TNM stage of the tumor is_______?

A. T1 N1 Mo
B. T2 N2 Mo
C. T3 N1 Mo
D. T4 N2 Mo

Tumors metastatic to the jaw bones are most likely to be found in the _______?

A. Mandibular condyle
B. Posterior moral region
C. Anterior maxilla
D. Maxillary tuberosity

Which of the following is an oral precancer ?

A. Oral hairy leukoplakia
B. White spongy naevus
C. Leukemia
D. Speckled Leukoplakia

White, spongy, folded thick mucosa is seen in______?

A. Oral hairy leukoplakia
B. Aspirin burn
C. White spongy nevus
D. Pseudomembranous Candidiasis

The condition involved with an unerupted tooth or impacted tooth is ________?

A. Dentigerous cyst
B. Mural ameloblastoma
C. Adenomatoid odontogenic tumor
D. All of the above

Ossifying fibroma manifests as______?

A. Unilocular radiolucency
B. Unilocular radio opacity
C. Multilocular radiolucency
D. Multiocular radio opacity
E. Both A & B

Which of the following conditions is least likely to present as an eccentric osteolytic lesion _____?

A. Aneurysmal bone cyst
B. Giant cell tumour
C. Fibrous cortical defect
D. Simple bone cyst

Peripheral giant cell granuloma occurs most commonly on the________?

A. Gingiva
B. Alveolar ridge
C. Palate
D. Floor of the mouth

The most common bone tumor that occurs in children is______?

A. Osteosarcoma
B. Ewing’s sarcoma
C. Metastatic carcinoma
D. Multiple myeloma

Hamartoma is _____?

A. Neoplastic
B. Non-neoplastic
C. Afflicted by trauma
D. Hormonal distubance

The tissue of which lesion has been described as resembling a blood sponge with large pores_______?

A. Cavernous hemangioma
B. Capillary hemangioma
C. Aneurysmal bone cyst
D. Eruption hematoma

Arecanut chewing is aetiological factor in______?

A. Leukoedema
B. Oral dubmucous fibrosis
C. Erythema multiforme
D. Oral lichen planus

Mobility of teeth in carcinoma of Maxillary sinus is due to involvement by tumor of________?

A. Anterior wall of the sinus
B. Posterior wall of the sinus
C. Roof of the sinus
D. Floor of the sinus

In which one of the following perineural invasion in head and neck cancer is most commonly seen ?

A. Adenocarcinoma
B. Adenoid cystic carcinoma
C. Basal cell adenoma
D. Squamous cell carcinoma

All of the following malignancies metastasize except______?

A. Basal cell carcinoma
B. Adenocarcinoma
C. Squamous cell carcinoma
D. Melanoma

Small palpable mass elevated above the epithelial surface is______?

A. Papule
B. Macule
C. Plaque
D. Vesicle

Multiple punched out lesion are seen in_____?

A. Paget’s disease
B. Osteosarcoma
C. Ewing sarcoma
D. Multiple myeloma

Teratoma is best describad as______?

A. Namartous developmental manifestation consisting of three different layers
B. Occurs most commonly in stomach and lungs
C. Originates from mesodermal tissue
D. Associated with very painful condition

Onion Skin appearance of radiographs is seen in_______?

A. fibrous dysplasia
B. osteosarcoma
C. Ewing’s sarcoma
D. Chondrosarcoma

Which of the following is benign in nature________?

A. Lymphoma
B. lymphangioma
C. Melanoma
D. Leukemia

On clinical examination a 60 years old female had a tumor in the right buccal mucosa. The size of the tumor was about 2 cm in diameter. There was no involvement of regional lymph nodes and also had no distant metastasis The TNM stage of the tumor is_____?

A. T1 No Mo
B. T1 NI Mo
C. T1 N2 Mo
D. T2 No Mo

Tobacco usage has been associated with______?

A. Hyperkeratosis
B. Erythema
C. Ulceration
D. None of the above

Acanthosis is_______?

A. Increase in mitotic division
B. Increase in thickness of superficial layer
C. Increase in thickness of spinous layer
D. Disruption of basal lamina

Melanoma is______?

A. Benign tumour of skin and mucous membrane
B. Malignant tumour of skin and mucous membrane
C. Malignant tumour of melanophores
D. Benign tumor of melanophores

Tumour that doesn’t show spontaneous regression_______?

A. Neuroblastoma
B. Retinoblastoma
C. Malignant melanoma
D. Octeosarcoma

Swelling and redness of the orifices of minor salivary glands of the palate occurs in______?

A. Nicotinic stomatitis
B. Leukoplakia
C. Fovea Palatine
D. Erythroplakia

A patient complains of numbness of lower lip. There is no history of tooth extraction. It could be________?

A. Infection
B. Metastatic neoplasia
C. Dental Manipulation
D. CNS disease

Carcinoma of the cheek_____?

A. CHaracteristically is columner-celled:
B. Has a recognized association with chewing gum
C. Has a recognized association with chewing betel nut
D. Has a recognized association with geographic tongue

Best results are obtained in oral submucous fibrosis is by_______?

A. Intralesional cortisone therapy. with hyaluronic acid
B. Oral cortisone with intralesional therapy (cortisone)
C. Vitamin E and oral cortisone
D. Intralesional placental extract

Presence of epithelial Pearls in spinous layer of epithelium is characteristic of_______?

A. Carcinoma
B. Pre-cancer
C. Dysplasia
D. Metaplasia

MOst common route for metastasis of oral cancer is by_______?

A. Direct textension
B. Lymphatics
C. Blood vessels
D. Aspiration of tumor cells

A man who had suffered from displaced fracture of mandible complains of pain in-old fracture site near mental foramen. There is a movable tender mass on palpation in the area The best preoperative diagnosis is_______?

A. Neuroleiomyoma
B. Neurofibroma
C. Trigeminal neuralgia
D. Traumatic neuroma

Hodgkin’s disease is considered to be_______?

A. Follilcular reticulosis
B. Inflammatory disease
C. Chronic granulomatous disease
D. A malignant neoplasm

White rough pedunculated lesion on palate is most likely________?

A. Pleomorphic adenoma
B. Papilloma
C. Nevus
D. Fibroma

Which of the following is a reactive lesion of the gingiva that may demonstrate bone radiographically and often even microscopically______?

A. Osteoma
B. Peripheral ossifying fibroma
C. Traumatic neuroma
D. Irritation fibroma

The most common bone cancer is_____?

A. Osteosarcoma
B. Metastatic bone cancer
C. Multiple myeloma
D. Squamous cell carcinoma

Persons with the greatest risk of oral cancer_______?

A. Have a poor oro-dental hygiene, nutritional deficiencies and are chronic alcoholics
B. Are middle aged have a poor oro-dental hygiene and regularly use tabacco
C. Are old, chronic alcoholics and regularly and regularly use tobacco
D. Are young, have poor oro-dental hygiene and are heavy smokers

Steriform pattern of fibrous tissue is seen in ________?

A. Fibrosarcoma
B. Malingant fibrous histiocytoma
C. Nerufibroma
D. Ameloblastic fibroma

A 3 cm squamous cell carcinoma of the retromolar trigone and invading the mandible and the medial pterygold muscle is at what TNM stage_______?

A. Stage I
B. Stage II
C. Stage III
D. Stage IV

Papillomatous tongue is observed in______?

A. Lymphangioma
B. Hyalinia cutus et mucosa syndrome
C. Fetal face syndrome
D. Tuberous scterosis

Metastases from carinoma of tongue by blood stream is more likely when the carcinoma involves the_____?

A. anterior third
B. middle third
C. posterior third
D. lateral margin

Features characteristic of leukoplakia include all except_______?

A. Hyperkeratosis
B. Plasma cell infiltration within the dermal papilae
C. Clinically, a paint like patch
D. A moist shiny lesion

Leukoplakia with the worst prognosis is seen on the_____?

A. Dorsum of tongue
B. Floor of mouth
C. Buccal mucosa
D. Palate

In which of the following type of tumour the HPV-6 can be detected ?

A. Papilloma
B. CEOT
C. Sarcoma
D. Pyogenic Granuloma

Which of the following epithelial changes commonly signify precancerous condition________?

A. Dyskeratosis
B. Hyperkeratosis
C. Parakeratosis
D. Acanthosis

Which of the following is not a feature of torus mandibularis ?

A. Common in Mongoloids
B. Present on the lingual surface of mandible below the mylohyoid line
C. Usually Bilateral
D. May or may not associated with torus palatinus

Which of the following does not have a viral etiology ?

A. Burkitt’s lymphoma
B. Nasopharyngeal carcinoma
C. Hodgkin’s lymphoma
D. Hepatocellular carcinoma

Increased incidence of squamous cell carcinoma of the skin is due to all except_____?

A. Ultraviolet radiation
B. Actinic keratitis
C. Alcohol
D. None of the above

Kaposi’s sarcoma is a tumour of______?

A. Blood vessels
B. Reticuloendothelial system
C. Striated muscles
D. Smooth muscles

Which of the following is most likely to be fatal ?

A. Osteochondroma
B. Giant cell tumour
C. Paget’s disease
D. Multiple myeloma

The most common malignancy of the oral cavity is_______?

A. Basal cell carcinoma
B. Transitional cell carcinoma
C. Melanoma
D. Squamous cell carcinoma

Which of the following conditions is characterized by cafe-au-lait spots, non-encapsulation and potential for malignant transformation_______?

A. Neurilemmoma
B. Neurofibroma
C. Traumatic Neuroma
D. Solitary plasmocytoma

Which of the following is NOT true of sturge Weber syndrome ?

A. Hemangiomatous involvement of skin
B. Mostly bilateral
C. Port wine nevus
D. Vascular gingival hyperplasia

Hemosiderin pricticles are seen histologically in case of______?

A. Fibroma
B. Pregancy tumor
C. Peripheral giant cell granuloma
D. Papilloma

Which of the following viruses are not implicated in human malignancies_______?

A. Epstein-Barr virus
B. Herpes simplex virus type I
C. HTLV 1
D. Papilloma virus

Which of the following has been implicated due to sunlight______?

A. Basal cell carcinoma
B. Lymphoepithelioma
C. Junctional nevus
D. Verruca vulgaris

During oral examination of a 57-year old man a large keratotic patch that covers the entire palate is noted some Red Spots are also seen in the patch The patient most likely is a_____?

A. Pipe smoker
B. Cigar smoker
C. Snuff chewer
D. Tobacco chewer

Satellite lesion with locally invasive property is seen in _______?

A. Chronic hypertrophic candidiasis
B. leukoplakia
C. dental ulcers
D. Hemangioma

On stretching the cheek the lesion disappears in______?

A. Leukoplakia
B. Focal hyperkeratosis
C. Leukoedema
D. Typhoid

Abtropfing affect is seen in______?

A. Junctional nevus
B. Pemphigus
C. Apthous ulcer
D. Erythema multiformae

Bowen’s disease is______?

A. Benign neoplasm of the G.I.T
B. Intro epithelial carcinoma
C. Vesiculobullous lesion of skins
D. Ulcerative lesion of G.I.T

Warty or Cauliflower like growth is_____?

A. Papilloma
B. fibroma
C. Lipoma
D. Torus

Bence- Jones are laboratory finding of_____?

A. paget’s disease
B. Multiple myeloma
C. Malignant melanoma
D. Fibrous dysplasia

Which of the following is a true cyst ?

A. Aneurysmal bone cyst
B. Gingival cyst of the newborn
C. Haemorrhagic bone cyt
D. All of the above

A rhabdomyoma is a tumour origination from______?

A. Never tissue
B. Smooth muscle
C. Striated muscle
D. Vascular endothelium

Which of the is the most common site for the occurrence of a basal cell carcinoma ?

A. Buccal mucosa
B. Hard Palate
C. Skin of the lower lip
D. Dorsum of the tongue

An Inflammed capillary hemangioma of the oral cavity looks similar to a _______?

A. Nevus
B. Neurofibroma
C. Pyogenic granuloma
D. Angiosarcoma

Numbness of lip seen with no previous dental treatment _______?

A. Meta static carcinoma
B. Central nervous system lesion
C. Osteomyelitis
D. Infection

In which of the following conditions pulsations or murmur may be detected _______?

A. Capillary hemangioma
B. Osteogenic sarcoma
C. Epidermoid carcinoma
D. Osteoid osteoma

Which of the following statement is true ?

A. Kaposi sarcoma is a benign tumour
B. Kaposi sarcoma is found in HIV positive subjects
C. Kaposi sarcoma is malignant fast growing tumour
D. Kaposi sarcoma is an ectodermal derivative

Hemosiderin pricticles are seen histologically in case of______?

A. Fibroma
B. Pregancy tumor
C. Peripheral giant cell granuloma
D. Papilloma

Warty or Cauliflower like growth is_______?

A. Papilloma
B. fibroma
C. Lipoma
D. Torus

Bence- Jones are laboratory finding of_____?

A. paget’s disease
B. Multiple myeloma
C. Malignant melanoma
D. Fibrous dysplasia

Which of the following is a true cyst ?

A. Aneurysmal bone cyst
B. Gingival cyst of the newborn
C. Haemorrhagic bone cyt
D. All of the above

A rhabdomyoma is a tumour origination from______?

A. Never tissue
B. Smooth muscle
C. Striated muscle
D. Vascular endothelium

Which of the is the most common site for the occurrence of a basal cell carcinoma ?

A. Buccal mucosa
B. Hard Palate
C. Skin of the lower lip
D. Dorsum of the tongue

An Inflammed capillary hemangioma of the oral cavity looks similar to a ______?

A. Nevus
B. Neurofibroma
C. Pyogenic granuloma
D. Angiosarcoma

Numbness of lip seen with no previous dental treatment _______?

A. Meta static carcinoma
B. Central nervous system lesion
C. Osteomyelitis
D. Infection

In which of the following conditions pulsations or murmur may be detected ______?

A. Capillary hemangioma
B. Osteogenic sarcoma
C. Epidermoid carcinoma
D. Osteoid osteoma

Which of the following statement is true ?

A. Kaposi sarcoma is a benign tumour
B. Kaposi sarcoma is found in HIV positive subjects
C. Kaposi sarcoma is malignant fast growing tumour
D. Kaposi sarcoma is an ectodermal derivative

Hemosiderin practicals are seen histologically in case of_______?

A. Fibroma
B. Pregancy tumor
C. Peripheral giant cell granuloma
D. Papilloma

Phleboliths are seen in_____?

A. Dental pulp
B. Salivary Gland duct
C. Cavernous Hemangiomas
D. Subepithelial Bulla

White-lesion with definite precancerous potential ________?

A. Leukoplakia
B. Discoid lupus Erythematosus
C. Lichen Planus
D. All of the above

Squamous papiloma is induced by ________?

A. EBV=
B. HSV
C. HPV=
D. CMV

Which of the following statements is true for MEN type ?

A. There is hyperplasia of parathyroid gland
B. Chromophobe adenoma of pituitary gland may result in acromegaly
C. Pancreatic tumours may produce gastrin, insulin glucagon, samatostain
D. All of the above

Benign tumour of voluntary muscle ____?

A. Leiomyoma
B. Rhabdomyoma
C. Rhabdomyosarcoma
D. Leiomyosarcoma

Port wine stain in characteristic feature of ________?

A. Peutz Jegers Syndrome
B. Sturge Weber Syndrome
C. Albright’s Syndrome
D. Lymphangioma

The most common benign tumor occurring in oral cavity is _______?

A. Papilloma
B. Fibroma
C. Adenoma
D. Epulis

Kaposi’s sarcoma is more commonly seen in patient with ________?

A. AIDS
B. Amyloidosis
C. Leukemia
D. HSV infection

Antoni type A and type B are seen in ________?

A. Neurofibroma
B. Neurilemmoma
C. Neurofibrosarcoma
D. Traumatic neuroma

Among pre-malignant oral lesions ______?

A. Leukoplakia should be proved by biopsy
B. Leukoplakia does not disappear even after cessation of smoking
C. Erythroplakia has a higher risk for malignancy than leukoplakia
D. Oral submucous fibrosis is seen in all parts of the world

A lesion composed of microscopic vessels is called as ______?

A. Haemangioma
B. Angioma
C. None of Haemangioma and Angioma
D. Heamangioma and Angioma

Which of the following is the diagnostic characteristic of peripheral giant cell granuloma ?

A. Mass of granulation tissue
B. Multinuclear giant cells
C. Keliod like enlargement
D. Epithelium is atrophic in some areas

In TNM classification T3 stands for tumour size _______?

A. >2 cm
B. >4 cm
C. 4 cm with invasion of adjacent structure

What could be the most appropriate provisional diagnosis for multiple nodular exophytic reddish lesions of oral mucosa in an AIDS patient ?

A. Hemangioma
B. Focal epithelial hyperplasia
C. Acute pseudo-membranous candidiasis
D. Kaposi’s sarcoma

Leiomyoma is a tumor of _____ ?

A. Cerebral tissue
B. smooth muscle
C. striated muscle
D. cardiac muscle

Which of the following is a pseudo cyst ?

A. Mucous retention cyst
B. Radicular cyst
C. Aneurysmal bone cyst
D. Dentigetous cyst

Metastatic disease to the oral region is most likely to occur in which of these locations ?

A. Tongue
B. Posterior maxilla
C. Posterior mandible
D. Floor of the mouth

An epithelial lining is typically found in all of the following except________?

A. Sinuses
B. Fistulae
C. Aneurismal bone cysts
D. Keratocytes

Cancer which most commonly metastasizes to jaw bone is_______?

A. Breast
B. Prostatic
C. Lung
D. Kidney

Osteosarcoma of the jaw_____?

A. Occurs mostly in the maxilla
B. Seen in old age
C. Highly malignant tumour which shows early metastasis
D. Shows a soap bubble type of radiolucency in radiographs

The most common malignant tumor of the gingiva is _____?

A. Malignant
B. Sarcoma
C. Fibro-sarcoma
D. squamous cell carcinoma

The most common site of metastasis from the mandibular sarcoma is______?

A. Lung
B. Liver
C. Spleen
D. Heart

All of the following are precancerous conditions except ______?

A. SLE
B. Peutz-Jeghers syndrome
C. Plummer-vinson syndrome
D. Xeroderma pigmentosum

Oral submucous fibrosis is diagnosed by_____?

A. Juxta, epithelial fibrosis (changes)
B. Changes in epithelium
C. Changes in submucosa
D. All of the above

Presence of Verocay bodies and having predeliction for occurrence in the tongue are seen in ______?

A. Granular cell myoblastoma
B. Neurilemmoma
C. Neurofibroma
D. Metaplasia

Which of the following is most malignant?

A. Neurolemmoma
B. Neurofibroma
C. Neurogenic fibroma
D. Traumatic neuroma

Which on of the following is a connective tissue tumour _____?

A. Lipoma
B. Melanoma
C. Carcinoma
D. Papilloma

Which of the following is carcinoma of the skin spreads by local invasion and has no tendency to metastasise ?

A. Malignant melanoma
B. Basal cell carcinoma
C. Fibrosarcoma
D. Leukoplakia

Ressell’s bodies are found in_____?

A. Activated macrophages
B. Histiocytes
C. Plasma cells
D. Erythrocytes

Which of the following is untrue about malignant melanoma ?

A. occurs with equal frequency in both se*es
B. Rare in children
C. Palate is the most common intraoral site
D. Is very painful

Which among the following shows pesudo-epitheliometous hyperplasia________?

A. sq. cell carcinoma
B. Basal cell arcinoma
C. Verrucous carcinoma
D. Granular cell myoblastoma

Increased incidence of carcinoma is observed with_______?

A. Homogenous leukoplakia
B. Verrucous leukoplakia
C. Nodular leukoplakia
D. Ameloblastic fibroma

Asymmetric widening of the periodontal ligament around two or more teeth is seen in_______?

A. metastatic breast carcinoma
B. osteosarcoma
C. paget’s disease
D. Fibrous dysplasia

Commonly involved lymph nodes during metastasis from carcinomas of oral cavity is_______?

A. Jugulo omohyoid and jugulo digastric
B. sub mandibular and servical lymph nodes
C. submental and submandibular lymphnodes
D. None of the above

Most common site of oral leukoplakia is_____?

A. angle of mouth
B. cheek mucosa
C. Soft palate
D. Gingiva

A patient is diagnosed of oral cancer or stage T,N,M______?

A. Surgery
B. Surgery +radiotherapy
C. Chemotherapy alone
D. Surgery+ chemotherapy

Osteosarcoma presents a radiographic picture resembling______?

A. Sun-brust appearance
B. Cotton-wool appearance
C. Soap-bubble appearance
D. Ground-glass appearance

A 20yr old patient reports with multiple swelling of the jaws. Clinical examination reveals multiple hard swellings involving the jaws and intra orally several missing teeth are noticed. Panoramic radiograph reveals multiple radio opaque lesions in the maxilla and the mandible with multiple impacted teeth and supernumerary teeth indicative of Gardeners syndrome: The above mentioned condition is_______?

A. Autosomal dominant disease
B. Autosomal recessive disorder
C. X-linked disease
D. A syndrome of unknown cause

Hemangiopericytoma resembles_____?

A. Hemangioma
B. Glomous tumour
C. Ewings tumour
D. Plasmacytoma

Brachytherapy is_______?

A. Irradiation of tissues from a distance
B. Irradiation of tissues from a distance of 3 cm
C. Irradiation of tissues by implants within the tissues
D. Irradiation of tissues by radiopharmaceuticals

Sarcoma of the soft tissues spread by________?

Sarcoma of the soft tissues spread by:

A. Blood vessels
B. Lymphatics
C. Direct invasion
D. Local infiltration

Squamous cell carcinoma with best prognosis is_______?

  1. Lip
    B. Tongue
    C. Palate
    D. Floor of the mouth

Acanthosis with intraepithelial vacuolation and hyperparakeratosis is seen in_____?

A. Hairy tongue (lingua villosa)
B. Hyperplastic candidiasis
C. Speckled leukoplakia
D. Desquamative gingivitis

The sign or symptom most suggestive of metastatic disease is______?

A. Paraesthesia
B. Sudden swelling
C. Root resorption
D. Diffuse radiolucency

The oral mucosa becomes rigid, blanched and opaque in which of the following conditions?

A. Pemphigus vulgaris
B. Lupus erythematoses
C. Ehlers-Danlos syndrome
D. Submucous fibrosis

Oral hairy leukoplakia is seen in which of the following conditions ?

A. AIDS
B. Hepatitis B
C. Smoker’s keratitis
D. Candidiasis

A non-painful, slowly enlarging benign neoplasm appears as a submucosal lump and exhibits pseudoepitheliomatous hyperplasia in the overlying epithelium, is most probably_____?

A. Fibroma
B. Rhabdomyoma
C. Granular cell tumour
D. Papilloma

Commonest site for carcinoma of tongue is______?

A. Lateral margin
B. Tip of tongue
C. Posterio 1/3
D. Ventral surface

The most reliable single histologic criterion for diagnosis of oral squamous cell carcinoma is____?

A. Invasion
B. Degeneration
C. Pleomorphism
D. Encpsulation

The most common site for metastasis from carcinoma of cheek________?

A. Brain
B. Regional lymph nodes
C. Lung
D. Liver

The most common precancerous lesion for oral malignancy is_______?

A. chronic hypertrophic candidiasis
B. Leukoplakia
C. Dental ulcers
D. atrophic glossitis

1.     How to Join Pak Army Lady Cadet Corp? Click Here
2.     How to Join Pak Army through Direct Short Service Commission? Click Here
3.     How to Join Pak Army Medical Corps? Click Here
4.     How to Join Pak Armed Forces Nursing Service? Click here
5.     How to become M-Cadet in Pak Navy?  Click Here
6.     How to Join Pak Navy through Short Service Commission? Click Here

Pak Army Academic Test: Online Test Preparation of Oral Pathology & Medicine

Carcinoma of lip______?

A. occurs mostly is female
B. metastases easily
C. occurs mostly in the lower lip
D. is mostly basal cell type

Granular cell myoblastoma of tongue is_____?

A. Benign tumor
B. Malignant tumor of the tongue
C. Lymphatic enlargement of the tongue
D. Developmental anomaly of the tongue

Starry sky appearance is seen in______?

A. Pagets disease
B. Cherubism
C. Garrey’s osteomyelitis
D. Burkitts lymphoma

Which of the following conditions is not considered as premalignant_____?

A. Erosive lichen planus
B. Syphilitic glossitis
C. Leukoedema
D. Leukoplakia

Frequent bouts of epistaxis are a conspicuous features of_______?

A. Encephalotrigeminal angiomatosis
B. Hereditary haemorrhagic telangiectasia
C. Nasopharyngeal angiofibroma
D. Vascular nevus

The term poikilokaryosis refers to______?

A. Alteration of nuclear cytoplasmic ratio
B. Division of nuclei without division of cytoplasm
C. Large, Prominent nuclei
D. Loss of polarity and disorientation of cells

Which of the following is the most likely (among them) to turn malignant ?

A. Intradermal nevus
B. Junctional nevus
C. Lichen planus
D. Papilloma

Reed-sternberg cells are characteristically seen in____?

A. Alpha-thalassemia
B. Glandular fever
C. Hansan’s disease
D. Hodgkin’s disease

Which of the following produces osteoblastic secondaries_______?

A. Carcinoma lung
B. Carcinoma breast
C. Carcinoma urinary bladder
D. Carcinoma prostate

Hereditary hemorrhagic telangiectasia is seen commonly on______?

A. Lips
B. Buccal mucosa
C. Tongue
D. Palate

Which of the following lesions are seen in van recklinghausen’s disease of skin______?

A. Hemangioma
B. Ameloblastoma
C. Neurofibroma
D. Giant cell fibroma

Which of the following is false regarding measles ?

A. Koplik’s spots
B. Muculo papular skin rash
C. Fever and malaise
D. Nikolsky’s sign

Papulovesicular oral lesions are seen in________?

A. Measles
B. Herpangina
C. Rubelia
D. Hand foot and disease

A 60 year old male presents with a 2 day history of a severe left ear ache with a burning sensation in the ear and loss of taste. There is left sided weakness of both upper and lower facial muscles. Facial sensation is normal. A few vesicles are seen in the pharynx. What is the most likely diagnosis ?

A. Bell’s palsy
B. Acoustic neuroma
C. Diphtheria
D. Ramsay Hunt syndrome

The most common reported oral malignancy in HIV infection is______?

A. Squamous cell carcinoma
B. Verrucous carcinoma
C. Multiple myeloma
D. Kaposi sarcoma

The virus which causes Herpes zoster is same as that which causes______?

A. Herpes simplex
B. Herpangina
C. Chicken pox
D. Measles

The lesion which is erythematous, recurrent and distributed along with the sensory nerve trunk is_____?

A. Herpes zoster
B. Erythema multiformae
C. Herpetic gingivo stomatis
D. Recurrent aphthous

Rubeola refers to_______?

A. German measles
B. Measles
C. Small pox
D. Chicken pox

Which is best for diagnosis of primary herpes simplex infection ?

A. Smear stained with Giemsa stain
B. Smear stained with Wrist’s stain
C. Flurorescent staining of cytology smear
D. Routing cytology

A 40-year old lady presents with unilateral demotonic distribution of veslcular eruptions. associated with severe pain. The most likely diagnosis is________?

A. Herpes zoster
B. Chicken pox
C. Recurrent herpes simplex infection
D. infectious mononucleosis

Acute herpetic gingivostomatitis____?

A. occur in children and young adults
B. occur only in young children
C. occur in only adults
D. All of the above

Steroids are indicated in all of the following conditions except_______?

A. oedema following extractions
B. oral ulcers in AIDS patients
C. TMJ arthritis
D. Angioneurotic oedema

Shingles occurs_______?

A. Primary infection
B. is unilateral
C. occurs on movable tissues
D. is bilatral

Cytological smear showing multinucleated giant cells, synctium and ballooning degeneration of the nucleus is a characteristic of______?

A. Herpes simplex virus infection
B. Erythema multiforme
C. Recurrent apthous stomatitis
D. Coxsackie virus infection

Herpetiformis vesicles, which rupture and leave areas of superficial intraoral ulcers, are caused by_____?

A. Varicella Zoster virus
B. Herpes zoster virus
C. Coxsackie virus
D. None of the above

Ramsay Hunt syndrome the cranial nerve involved is______________?

A. Trigeminal
B. Facial
C. Glossopharyngeal
D. Occulomotor nurve

In herpes primary lesion is______?

A. ulcer
B. papule
C. vesicle
D. Bullae

Mumps is caused by______?

A. Orthomyxo virus
B. Paramyxo virus
C. Rheno virus
D. EB virus

Primary herpetic lesions involving the gingiva are most likely to occur during ages______?

A. 1-5 years
B. 6-12 years
C. 13-16 years
D. They are likely to occur equally at any age

Recurrent ulcers occurring on gingiva and palate are most probably______?

A. Aphthous ulcers
B. Herpes simplex
C. koplick spots
D. Lesions of Behcet’s syndrome

Coxsackie virus causes____?

A. infectious mononucleosis
B. lymphoma
C. herpangina
D. herpes

A 3 year old child has a fever of 102 degree F; and following upper respiratory tract infection discrete vesicles and ulcers on the soft plate and pharynx are noted, The most probable diagnosis is _______?

A. Herpangina
B. Scarlet fever
C. Rubella
D. Herpetic gingivostomatitis

Which of the following would be seen in late HIV cases ?

A. Kaposi’s sarcoma
B. Oral hairy leukoplakia
C. Gingivitis/periodontitis
D. All of the above

Which of the following does not cause oral cancer in children ?

A. Herpes simplex
B. Esptein bar
C. Cytomegalovirus
D. Varicella zoster

Which of the following is not associated with gingival lesions ?

A. Herpes
B. Recurrent apthae
C. Pemphigus
D. Pyogenic granuloma

In HIV, which cells are affected_______?

A. CD4
B. CD8
C. Monocytes
D. Lymphocytes

Which virus given below is not a teratogenic virus ?

A. Rubella
B. Cytomegalovirus
C. Herpes simplex
D. Measles

Herpes simplex is seen in____?

A. < 10 yrs. of age
B. 12-15 yrs. of age
C. 25-30 yrs. of age
D. 55-60 yrs. of age

Which of the following is NOT True about primary HSV infection ?

A. primarily affects the anterior portion of the mouth causes acute gingivits
B. causes acute gingivits
C. occurs as epidemic
D. shows prodromal symptoms

Herpangina is caused by______?

A. Herpes simplex virus
B. Coxsackie virus
C. Measles virus
D. Varicella zoster virus

Coxsackie virus is implicated in______?

A. Herpes zoster
B. Measles
C. Small pox
D. Hand, Mouth & foot disease

Drug used in case of Herpetic lesions_______?

A. Acyclovir
B. Penicillin
C. Tetracycline
D. Ciprofloxacin

Recurrent herpes occurs due to______?

A. Virus in oral mucosa
B. Latent virus is skin supplying the area
C. Latent virus in nerve ganglia
D. None of the above

Site specificity is seen in_______?

A. Syphilis
B. Recurrent herpes labialis
C. Carcinoma
D. Traumatic aphthous ulcer

Reactivation of varicella virus in a posterior root ganglion results in_______?

A. chicken pox
B. Herpes zoster
C. Herpes simplex
D. Poliomyelitis

Which of the following is a difference between herpangina and primary herpetic stomatitis_______?

A. It is preceded by prodromal symptoms
B. It is unilateral in nature
C. Ulcers relationship seen on the anterior faucial pillars
D. Viral etiology

Prodromal symptoms precede 1 to 2 days before the onset of disease in_______?

A. Viral fever
B. erythema multiforme
C. pemphigus
D. pemphigoid

All about herpangina are false except_______?

A. May lead to high grade fever
B. May cause Dysphagia
C. May lead to vesicles formation in Anterior part of mouth around 20-30 in number
D. Anterior portion of mouth have only minor vesicles

Which of the following medications shortens the recovery period of primary Herpetic gingivostomatitis ?

A. Acyclovir
B. Ziduvidine
C. Kenalog in orabase
D. All of the above

The reason why most patients suffering from recurrent herpes labialis rarely give a history of having acute form of the herpetic gingivastomatitis is because______?

A. Etiological agents differ
B. The acute form occurs only inversely immuno compromised individuals
C. The primary infection was subclinical
D. The patient has received antibodies during intrauterine life and the antibodies have persisted

Inflammation of the dorsal root ganglion and vesicular eruption of the skin and mucous membrane in area supplied by a sensory nerve that is affected in characteristic of_____?

A. Herpes zoster
B. Herpes simplex
C. Uveoparotid fever
D. Aphthous stomatitis

Which of the following is associated with HIV infection______?

A. Hairy leukoplakia
B. Erythroplakia
C. Oral lichen planus
D. Bullous pemphigoid

Koplik spots______?

A. First manifestation of measles
B. Rarely seen in measles
C. Are seen 2-3 days after cutaneous rashes
D. Is first manifestation but seldom seen

Intro nuclear inclusions detected during the course of herpes simplex virus infection are called______?

A. Bacteriophages
B. Lipschutz bodies
C. Negri bodies
D. Donavan bodies

The feature that distinguishes herpes zoster from other vesiculo bullous eruption is______?

A. Unilateral occurrence
B. Severe burning pain
C. Prominent crusting vesicles
D. Sub epidermal bullous formation

Which of the following occurs most commonly on tongue ?

A. Lymphangioma and granular cell myoblastoma
B. Lipoma and fibroma
C. Neuroblastoma and lipoma
D. Lymphangioma and fibroma

In amyloidosis of the tongue, The amyloid is deposited primarily in the_____?

A. Stromal connective tissue
B. cells of the surface epithelium
C. Nuclei of the striated muscle cells
D. Cytoplasm of the striated muscle cells

squamous cell carcinoma on tongue most common site is____?

A. Apex
B. Base
C. Lateral borders
D. Dorsum

Median rhomboid glossitis is associated with____?

A. Burning mouth syndrome
B. Fungal infection
C. Bacterial infection
D. Absence of filiform papillae

A median round lesion in front of circumvallate papilae, with epithelial hyperplasia diagnosis is___________?

A. Median rhomboid glossitis
B. Erythema migrans
C. Apthous ulcer
D. Chemical brun

Hairy Tongue is a condition in which certain structures become enlarged______?

A. Fungiform papillae
B. Filiform papillae
C. Circumvallate papillae
D. Taste buds

Leutic glossitis can occur in connection with_______?

A. Vitamin B deficiency
B. Vitamin C deficiency
C. Iron deficiency
D. Syphilis

The syndrome of geographic tongue should be treated by______?

A. Excision of the lesion
B. Penicillin therapy
C. Topical application of Nystatin
D. Routine observation at recall time

Magenta tongue is found in the deficiency of the vitamin_____?

A. Riboflavin
B. thiamine
C. nicotinic acid
D. pyridoxine

Glossodynia is_____?

A. Pain in the tongue
B. Burning of the tongue
C. Swelling of the tongue
D. White patch on tongue

Oral hairy leukoplakia is seen in AIDS patients. The most likely site of appearance is______?

A. Lateral borders of tongue
B. Sublingual muosa
C. Soft palate
D. Buccal mucosa

Following a general examination and a biopsy of a firm, pale nodule in the tongue, a diagnosis of primary amyloidosis has been reached What underlaying disease is this patient likely to have_______?

A. Leprosy
B. Syphilis
C. Tuberculosis
D. None of the above

Burning sensation of the tongue is called_____?

A. Glossopyrosis
B. Glossodynia
C. Glossitis
D. Glossolgia

Median rhomboid glossitis is due to_____?

A. Inflammation of the tonge
B. Persistence of tuberculum impar
C. Hypertrophy of filiform papillae
D. Atrophy of filiform papillae

A blue nodular mass on the lateral border of the tongue is soft, smooth and blanches upon pressure It is most likely to be______?

A. Lymphoma
B. Hemangioma
C. Epulis fissuratum
D. Epithelioma

Acquired, symmetric hyper pigmentation of the sun exposed skin of the face & neck which is strongly associated with pregnancy & use of oral contraceptives is called as_______?

A. Melanoma
B. Cafe-au-lait-spots
C. Freckle
D. Melasma

Pink’s disease is due to______?

A. Toxicity of silver
B. Toxicity of Mercury
C. Toxicity of lead
D. Toxicity of Tetracycline

Cafe-au-lait spots on the skin are characteristic of______?

A. Addision’s disease
B. Peutz-Jeghers syndorme
C. Von recklinghausen disease
D. Hyper pituitarism

Green discoloration of teeth is seen in_______?

A. Tetracycline therapy Fluorosis
B. Fluorosis
C. Erythroblastosis fetalis
D. None of the above

In peutz-Jeghers syndrome, the oral lesions are_______?

A. Ulcerations of oral mucosa
B. Sebaceous glands of oral mucosa
C. Silver pigmentation of oral mucosa
D. None of the above

A patient showing brownish pigmentation and with normal laboratory findings may be suffering from____?

A. Addison’s disease
B. fibrous dysplasia
C. Neuro fibromatosis
D. None of the above

Green stains which occur frequently in children are due to_______?

A. Enamel deficiency
B. Dentin deficiency
C. Material alba
D. Chromogenic bacteria

Disease which increase oral melanin pigmentation_______?

A. addison’s disease
B. Hyperthyroidsim
C. Nephritis
D. All of the above

The most common intraoral location for a pigmented nevi is the_____?

A. Hard palate
B. Soft palate
C. Buccal mucosa
D. Floor of mouth

Melanin pigmentation in pregnancy is known as_____?

A. Melasma
B. Melanoma
C. Epulis
D. Melanosis

The common site of melanoma on the orofacial skin is______?

A. Lower lip
B. Malar region
C. Forehead
D. Upper lip

Yellowish discolouration of oral mucous membrane, skin and sclera of eye is______?

A. Pernicious anemia
B. Sickle cell anemia
C. Chloromycin therapy
D. Carotinemia

All of the following are seen in lead poisoning except_____?

A. Hallucinations
B. GIT disturbances
C. Peripheral neuritis
D. Encephalitis

Mucocutaneous circumoral pigmentation is found in______?

A. Peutz-Jeghers syndrome
B. Plummer-Vinson syndrome
C. Lead poisoning
D. Bechet’s syndrome

Tetracycline stains appear as______?

A. Yellow and brown stains in enamel and dentin
B. Yellow and brown stains only in enamel
C. Yellow and brown stains only in dentin
D. Only yellow stain in enamel

Port wine stains are seen in_____?

A. Nevus
B. Haemangioma
C. Melanoma
D. All of the above

Patient reports with discolored teeth bearing brown stains. The teeth glow fluorescent in UV light. The most likely diagnosis is______?

A. Porphyria
B. Amelogenesis imperfecta
C. Hutchinson’s teeth
D. Tetracycline staining of teeth

Yellowish discoloration of teeth is seen in children fed on_____?

A. High-protein diet
B. Tetracyclines
C. penicillins
D. Erythromycin

The pigment associated with hemochromatosis is______?

A. Biliribin
B. Haemosiderin
C. Methemoglobin
D. Myoglobin

Internal resorption is characterized by_______?

A. pain on percussion
B. Slow dull continuous pain
C. No characteristic feature, symptom free
D. Increased pulpal pain when laying down

Loss of tooth surface because of chemomechanical action is known as_____?

A. Abrasion
B. Erosion
C. Abraction
D. Attrition

Sloughing of necrotic epithelium is characteristic of________?

A. Aspirin burn
B. Denture sore mouth
C. Traumatic ulcer
D. Contact dermatitis

Perleche is caused by______?

A. decreased interdental space
B. increased interdental space
C. Lack of Vit-C
D. Trauma to the corners of the mouth

Bismuth intoxication is manifested as_____?

A. pigmentation is areas of inflammation
B. Burtonian line
C. Gingiva
D. Blackish line in the mucosa

Silver tatoo is due to_____?

A. deposition of Ag Amalgam in Mucosa
B. deposition of Ag Amalgam in Bone
C. deposition of Ag Amalgam in Dentin
D. deposition of Ag Amalgam in Enamel

In a 2 year old child, a bluish dome shaped swelling on the inner side of the lip is mostly______?

A. Hematoma
B. Mucocoele
C. Hemangioma
D. None of the above

Mucooceles are rarely seen in the_____?

A. Lower lip
B. Upper lip
C. Buccal mucosa
D. Floor of the mouth

Eighty percent of all salivary stones occur in_____?

A. Parotid
B. Submandibular
C. Sublingual
D. Minor salivary glands

Enlargement of lip occurred with in seconds to 24 hours is______?

A. Angioedema
B. Mucocoele
C. Herpes
D. Fibroma

Angular stomatitis______?

A. can be a concomitant of any candidal infection
B. is seen mainly in adults
C. may be associated with streptococcus
D. may be a sign of anaemia

Cyst without lining is_____?

A. Radicular cyst
B. Dentigerous cyst
C. Naso palatine duct cyst
D. Hemorrhagic / Traumatic bone cyst

The etiology of angioedema is_____?

A. Sensitivity to acrylic resin
B. Using ACE inhibitors
C. Hereditary
D. All of the above

A patient with emotional problems, increased salivation; pallor of oral mucosa and a grayish blue discoloration of the gingiva. These findings are most consistent with a clinical impression of____________?

A. Cherubism
B. Cretinism
C. Pierre Robin Syndrome
D. Lead poisoning

Hyperpigmentation is seen in all except_____?

A. peutz jeghers syndrome
B. addison’s disease
C. cushing’s syndrome
D. albright syndrome

Increase in height of mandible with increase in interdental spaces in elderly man______?

A. Hyperpituitarism
B. Hyperthyroidism
C. Hypopituitarism
D. Hyperparathyroidism

Wernick’s encephalopathy is caused by deficiency of_______?

A. Thiamine
B. Cyanocobalamine
C. Niacin
D. Riboflavin

A 9 year old child has increased horizontal anterior bone loss, cementum and on test shows excretion of phophoethanolamine in the urine. The child is suffering from______?

A. Hypophosphatasia
B. Vit. D resistant Rickets
C. Juvenile periodontitis
D. Osteomalacia

Which of the following vitamin in associated with manifestations of neurological problem_______?

A. Vit A
B. Vit K
C. Folic acid
D. Cyanacobalamin

Pigmentation occurs in oral cavity and skin in all of the following except_______?

A. Peutz-jeghers syndrome
B. Addision’s syndrome
C. Cushing syndrome
D. Albright syndrome

65 year old man who is anaemic, complaints of back pain multiple radiolucencies in panaromic view_____?

A. Multiple myeloma
B. Osteosarcoma
C. Giant cell granuloma
D. Eosinophilic

Intestional absorption of calcium is decreased by_______?

A. proteins
B. lactose
C. phytic Acid
D. Acidity

Hypogonadism developmental delay, loss of taste and smell is due to deficiency of________?

A. Cu
B. Zn
C. K
D. Cr

Macrodontia is associated with_______?

A. Acromegaly
B. Pituitary gigantism
C. Hypoparathyroidism
D. Hyperthyroidism

Beefy red and painful tongue is characteristic of______?

A. Vitamin A deficiency
B. Any periodontal disease
C. Acute nicotinic acid deficiency
D. Ascorbic acid deficiency

Consider the following statements giant cells are a characteristic histopathologic finding in______?

A. Apthous ulcers
B. Keratocyst
C. Brown tumor of hyper parathyroidism
D. Dentigerous cyst

Which of the following is associated with a low concentration of ionized calcium in the serum ?

A. Hypothyroidism
B. Osteogenesis imperfecta
C. Paget’s disease of the bone
D. Tetany

An abnormal resorption pattern in primary teeth, delayed eruption of permanent teeth and a large tongue are the feature of______?

A. Addison’s disease
B. Hypothyroidism
C. Hyperthyroidism
D. Von-Recklinghausen disease

Acrodermatitis enteropathica is due to deficiency of______?

A. Mercury
B. Zinc
C. Lead
D. Bismuth

Bone changes in hyperparathyroidism include______?

A. Generalized demineralization
B. Brown tumours
C. Cystic changes
D. All of the above

A disease which only affects the formation and eruption of tooth but does not cause hypoplasia is______?

A. Hypoparathyroidism
B. Hyperthyroidism
C. Rickets
D. Syphilis

The histiocytosis X is a spectrum of disorders, Which include the following condition ?

A. Eosinophilic granuloma
B. Hand Schuller – Christian disease
C. Letterer – Siwe disease
D. All of the above

Magenta tongue and cracks at corner of mouth are seen is deficiency of______?

A. Vitamin B
B. Niacin
C. Riboflavin
D. Pantothenic acid

Which of the following may be a feature of acromegaly ?

A. Large tongue
B. Micrognathia
C. Hypoglycemia
D. Crowded teeth

Oral lesions on the tongue and other mucosal surfaces of the oral cavity is caused by the deficiency of_______?

A. Vit – B1
B. Niacin
C. Vit – C
D. Vit – K

A progressive increase in mandibular length and in mandibular interdental spacing in an adult patient is characteristic of______?

A. Periodontosis
B. Hypothyroidism
C. Hyperpituitarism
D. Hypoadrenalism

Which of the following is least affected in Vit – C deficiency_____?

A. Gingival Fibres
B. Periodontal ligament
C. Blood Vessels of the gingiva
D. Epithelial lining of the mucosa

Which of the following tooth structure during formation is most effected due to Vit – A deficiency______?

A. Enamel
B. Dentin
C. Cementum
D. Periodontal Ligament

Osteomalacia is______?

A. Defective osteoid+ normal mineralization
B. normal osteoid + defective mineralization
C. abnormal osteoid + abnormal mineralization
D. normal osteoid an demineralization

A five-year old child presents with chronic bed wetting and bilateral loose deciduous first molars. His mother says that she want to drink of water several times during each night. Which of the following is the most likely diagnosis ?

A. Hand-Schuller Christian disease
B. Marble bone diasease
C. Nieman pick disease
D. Polyostotic fibrous dysplasia

Addison’s disease typically_____?

A. Causes hypertension
B. causes hypopigmentation
C. Is an autoimmune disease
D. Steroids are contraindicated

A histologic evidence of widespread formation of globular hypocalcified dentin and pulp horns reaching the dentinoenamel junction, absence of lamina dura around the tooth in radiograph are the characteristic features of_______?

A. Vitamin -D resistant rickets
B. Hypophosphatasia
C. Hypervitaminosis-A
D. Vitamin-A deficiency

Reilly bodies are inclusion bodies seen in hurler’s disease within_____?

A. Lymphocytes
B. Fibroblast
C. RBC
D. WBC

Hepatolenticular degeneration is seen with deposition of_______?

A. cadmium
B. lead
C. aluminium
D. copper

Which of the following may be a feature of acromegaly ?

A. Large tongue
B. Micrognathia
C. Hypoglycaemia
D. Crowded teeth

All the following affects absorption of calcium EXCEPT______?

A. citric acid
B. Retinoic acid
C. phytates
D. Oxalates

Hyperparathyroidism radiological features seen are_____?

A. Loss of laminadura
B. Osteitis fibrosa cystica
C. Erosion below the duramater of skull
D. All of the above

Normal serum calcium level is_____?

A. 5 to 7 mg%
B. 7 to 9 mg%
C. 9 to 11 mg%
D. 11 to 13 mg%

Brown tumours are seen in_____?

A. Hyperparathyroidism
B. Pigmented villonodular synovitis
C. Osteomalacia
D. Neurofibromatosis

Swollen joint, anemic, loose teeth & dentin dysplasia are because of deficiency of_____?

A. Vitamin C
B. Vitamin D
C. Vitamin B
D. Vitamin E & D

Red fluorescent fluid is seen in____?

A. Pemphigus
B. Erythema multiforme
C. Lichen planus
D. prophyria

Letterer seiwe disease is a disturbance of______?

A. protein metabolism
B. histiocytic disorder
C. Mucopolysaccharide metabolism
D. Carbohydrate metabolism

Bone pain, bone, cyst, fractures and renal stones are characteristics of________?

A. Hyperparathyroidism
B. Cushing’s syndorme
C. Multiple myeloma
D. Marfan’s syndrome

A 50 year old obese man complains of several recent abscesses in the gingiva with loosening of teeth. He also suffers from itching of skin and polyuria. The most probable etiology is_______?

A. Scurvy
B. Myxoedema
C. Diabetes mellitus
D. Vitamin A deficiency

Lamina Dura is lost of partially lost in____?

A. hypothyroidism
B. hyperthyroidism
C. hypoparathyroidism
D. hyperparathyroidism

Vitamin “D” deficiency causes all except____?

A. Widening of predentin
B. Defective calcification
C. Microdontia
D. Inter globular dentin formation

Eosinophilic granuloma results from the proliferation of______?

A. Histiocytes
B. eosinophils
C. Lymphocytes
D. Fibroblast

Premature exfoliation of deciduous teeth is seen in_____?

A. Hypophosphatasia
B. Hypophosphatemia
C. Hyper phosphtasia
D. Hyperparathryroidism

Diabetes insipidus, bone lesions lesions and exophthalmos is seen in______?

A. Sickle cell anemia
B. Niemen pick disease
C. Littere Siewe Disease
D. Hand-Schuller-Christian disease

Gingiva is most commonly affected by deficiency of____?

A. Vitamin A
B. Vitamin D
C. Vitamin C
D. Vitamin B

Deficiency of which of the following will cause enamel hypoplasia ?

A. Vitamin B and C
B. Vitamin A, C and D
C. Calcium
D. Vitamin C

Which of the following deficiencies are associated with the disorders of hyperplasia of salivary gland and keratinisation of the salivary gland_____?

A. Vit – A
B. Vit – B
C. Vit – C
D. Vit – K

Dinesh, a 24-year old male, complains of loose teeth in a single quadrant. His radiograph shows irregular bone loss and histopathology reveals eosinophils and histiocytes. The most probable diagnosis is______________?

A. Hand- Schuller-Christian disease
B. Paget’s disease
C. Osteoclastoma
D. Albright’s syndrome

Addison’s disease is related to______?

A. Adrenal medulla
B. Adrenal cortex
C. Post pituitary
D. Parathyroid gland

1.     

Pakistan Army Selection & Recruitment Centers

Click Here

2.     

Pakistan Navy Selection Centers

Click Here

3.     

Pakistan Air Force Information & Selection Centers

Click Here

Oral Pathology and Medicine MCQs with Answers

The deficiency of which of the following vitamins does not effect on tooth development______?

A. Vit – A
B. Vit – D
C. Vit – C
D. Vit – K

Which of the following is related to an enzyme deficiency and involves periodontal destruction around primary teeth_____?

A. Hypophosphatasia
B. Cyclic neutropenia
C. Juvenile Periodontitis
D. Papillion Lefevre syndrome

Delayed eruption of at least part of the dentition is a recognized feature of all of the following EXCEPT_____?

A. Rickets
B. Congenital hyperthyroidism
C. Cleidocranial dysplasia
D. Cherubism

Prolonged use of antibiotics in children can result in_____?

A. Necrotising ulcerative gingivtis
B. Candidiasis
C. Actinomycosis
D. Apthous ulcers

Oral thrush develops in infants at_____?

A. 6 days
B. 2-6 weeks
C. 10-12 weeks
D. 18 mouths

A patient is having severe mycotic infection diabetes mellitus and cellulitis is suffering with______?

A. Mycormycosis
B. Histoplasmosis
C. Candidiasis
D. None of the above

Thrush____?

A. Occurs only in elderly
B. Is caused by a gram -ve bacteria
C. Is seen clinically as a proliferative lesion
D. Responds to nystatin

Denture sore mouth is cased by_______?

A. Actinomyces
B. candida albicans
C. Blastomyces
D. None of the above

Candid infection is not seen in____?

A. Median rhomboid glossitis
B. Geographic tongue
C. Oral thrush
D. Denture stomatitis

phycomycosis is also called____?

A. Botryomycosis
B. Coccidiomycosis
C. phytomycosis
D. Mucormycosis

ID reaction is associated with______?

A. apthous ulcer
B. herpetic stomatitis
C. syphilis
D. candidiasis

Darlings disease is_______?

A. Histoplasmosis
B. Phycomycosis
C. Actinomycosis
D. Bleomycosis

Vascular involvement and thrombosis is seen in____?

A. Coccidiomycosis
B. Aspergillosis
C. Mucormycosis
D. Histoplasmosis

Non-septate hyphae with a tendency to branch at 90 degree angle is characteristic of______?

A. Mucor
B. Aspergillosis
C. Cryptococcus neoformans
D. Coccidioides immitis

Oral candidiasis is divided into primary and_______?

A. Secondary infections
B. Subsidiary infections
C. Subclinical infections
D. All of the above

The yeast which shows thick gelatinous capsule and positive for mucicarmine is_____?

A. Cryptococcus
B. Histoplasmosis
C. Blastomycosis
D. Paracoccidiomycosis

Which type of candidiasis is associated with leukoplakia is_____?

A. Acute pseudomembranous candidiasis
B. Acute atrohpic candidiasis
C. Chronic atrophic candidiasis
D. Chronic hyperplastic candidiasis

Candidiasis associated with Dentures is______?

A. Acute Pseudo Membranous
B. Acute Atrophic
C. Chronic Hyperplastic
D. Chonic atrophic

Candida infection is involved in______?

A. migrating geographic tongue
B. median rhomboidal glossitis
C. Prolonged tetracycline therapy
D. All of the above

Drug used to treat oral thrush_____?

A. Clobetasol
B. Co-trimoxozle
C. Miconazole
D. Penicillin

The microorganism most commonly cultured from a chronic bilateral ulcer at the corner of mouth______?

A. Mucor
B. Candida
C. Treponema
D. Aspergillus

Clinical diagnosis of candidiasis is confirmed by_____?

A. Characteristic odour
B. Demonstration of mycelia and spores in scrapping
C. Response to injection of vitamin B12
D. Demonstration of ray fungus in granules

White raised painless areas when on peeling exhibit painful erythematous areas in case of_______?

A. Oral thrush
B. Pemphigus vulagaris
C. Leukoplakia
D. Erythroplakia

A deficiency of which of the following cell can predispose to candidiasis_____?

A. Eosinophils
B. Macropages
C. Plasma cells
D. T Lymphocytes

Thrush refers to_____?

A. Acute atrophic candidiasis
B. Acute pseudomembranous candidiasis
C. chronic atrophic candidiasis
D. chronic hyperplastic candidiasis

White patch is seen on the buccal mucosa consisting of pseudomycelium and chalmydospores with desquamated epithelium adjacent to it, the patient is suffering from epithelium adjacent to it, the patient is suffering from_______?

A. histoplasmosis
B. cryptococcosis
C. candidiasis
D. coccidiomycosis

All of the following statement about thrush are true EXCEPT_____________?

A. It is caused by a gram-negative fungus
B. It is characterized by a plaque of proliferating epithelial and other cells
C. It is a complication of immunosuppression or systemic diseases
D. It can affect neonates in an epidemic fashion

A 9 year old child’s mother comes to dental clinic with the complaint of oral ulceration, fever and shedding of skin of palms and soles: she is giving history of premature shedding of teeth and increased sweating she is also giving one month history of using any new teething gel available in market. The child is suffering form____?

A. Acrodynia
B. Pemphigus vulgaris
C. Epidermolysis Bullosa
D. Erosive lichen planus

Characteristic Tram- line calcifications in skull radiographs is observed in________?

A. Cleidocranial dysostoses
B. Sturge – weber syndrome
C. Paget’s disease
D. Mc Cline – Albright syndrome

Perception of taste even in absence of stimuli is known as______?

A. Ageusia
B. Dysguesia
C. Cocoguesia
D. Phantoguesia

Enzymes which play an important role in calcification are_______?

A. Enolase & calcitonin
B. Alkaline phosphatase & catalase
C. Alkaline phosphatase & pyrophosphatase
D. Pyrophosphatase & carbonic anhydrase

Strength of collagen is due to_______?

A. Hydroxy glacine
B. Glycine
C. Proline
D. Hydroxproline

Bruxism is characterized by____?

A. Increased mobility of the teeth
B. Radiographic widening of the pdl
C. Morning pain in muscles
D. All of the above

Strawberry gingivitis is seen in______?

A. Wegener’s Granulomatosis
B. Scorbutic Gingivitis
C. Plasma cell Gingivitis
D. Leukemic Gingivitis

What is the term for the radio opaque area found at the root apex of young permanent teeth involved with chronic pulpitis ?

A. Apical cyst
B. Apical condensing osteitis
C. Chronic apical periodontitis
D. Stage one apical osteofibroses

Antischkow cells are present in all of the following conditions except______?

A. Sickle cell anaemia
B. Iron deficiency anaemia
C. Apthous ulcer
D. Herpes simplex

Difference between epithelium of oral cavity and cavity lining of cyst is ____-?

A. Stratum corneum
B. Stratum lucidum
C. Stratum germinativum
D. Stratum spinosum

Which is the most preferred route for drug administration in the management of chronic pain______?

A. Intrathecal
B. Oral
C. Subdermal
D. Intravenous

The Sensitive period for tetracycline induced discolouration in the permanent maxillary mandibular incisors and canines is______?

A. 3 months postpartum to 7th year of life
B. 4 months in utero to 3 months postpartum
C. 5 months in utero to 9 months postpartum
D. Birth to 7th year

Which is T cell tumor ?

A. Burkitt’s lymphoma
B. Mycosis fungiodes
C. Mantel cell leukemia
D. Hairy cell leukemia

The disorder characterized by craniosynostoses, craniofacial anomalies, severe symmetrical syndactyly (cutaneous and bony fusion) of hands and feet along with prexial syndactyly and variable soft form syndactyly ?

A. Carpenter syndrome
B. Crounzon Syndrome
C. Apert Syndrome
D. Down’s syndrome

HLA – B27 histocompatbility antigen is seen in_______?

A. Sjogren’s disease
B. Ankylosing spondylitis
C. Felty’s syndrome
D. Scleroderma

For primary herpes simplex, the diagnosis is made by which gene_______?

A. Culture with giemsa stain:
B. Culture with wright strain
C. Routing cytology
D. Flourescent stain for cytology

Burning tongue might not be associated with____?

A. Ranula
B. Diabetes mellitus
C. Pernicious anaemia
D. Local irritation

Which of the following is a non-destructive method of age estimation in adults ?

A. Assessment of root dentin translucency
B. Amino acid racemisation
C. Evaluation of tooth cementum annulations
D. Pulp to tooth ratio of canines

Tooth discolouration due to high bilirubin secretion is seen in the______?

A. Pink tooth of mummery
B. Ochronosis
C. Chlorodontia
D. Leong teeth

Out of syphilitic glossitis, plummer vinson syndrome Mikulicz’s syndrome and hepatitis A; which of these predispose to squamous cell arcinoma ______________?

A. Syphilitic glossitis and plummer vinson syndrome
B. Syphilitic glossitis and Mikulicz’s syndrome
C. Plummer vinson disease and hepatitis A
D. Hepatitis A and Mikulicz’s Syndrome

Perimolysis is______?

A. Tooth wear due to gastric secretion
B. Tooth wear due to bruxism
C. Tooth wear due to dentifrices
D. Peripheral blood cell destruction

Differential diagnosis of hypercementosis includes all of these EXCEPT ?

A. Cemental dysplasia
B. Cemental aplasia
C. Condensing osteitis
D. Focal periapical osteopetrosis

Which of the following is a virus induced eipthelial hyperplasia ?

A. Molluscum contagiosum
B. Focal epithelial hyperplasia
C. Squamous papilloma
D. All of the above

Cardiac condition requiring Antibiotic prophylaxis for infective Endocarditis ?

A. Coronary Heart Disease
B. Rheumatic Heart Disease
C. Cardiac Pacemakers
D. Hypertensive Heart Disease

Biopsy of a clinically suspicious lesion is negative. The most appropriate treatment is______?

A. Tell patient no malignancy
B. Repeat the biopsy
C. Observe the patient for twelve months
D. Observe the patient for three months

Disease characterized by insidious onset and by an absolute increased in number of circulating RBCs and in total blood volume ?

A. Leukopenia
B. Osler’s disease
C. Mediterranean disease
D. Aplastic anaemia

Tubular (Canalicular) adenoma occurs on_____?

Tubular (Canalicular) adenoma occurs on:

A. Upper lip
B. Palate
C. Lower lip
D. Glingiva

Alveolitis sicca dolorosa is otherwise known as______?

A. Trigeminal neuralgia
B. Sicca syndrome
C. Dry socket
D. Myospherulosis

A radiograph of the mandibular anterior teeth in a patient reveals radiolucencies above the apices of right lateral and central incisors. No restorations or cavities are present There is no pain or swelling and the pulps are vital. The diagnosis is______?

A. Periapical granuloma
B. Cementoblastoma
C. Radicular cyst
D. Chronic abscess

Endocarditis prophylaxis is recommended during following dental procedures except______?

A. Dental extractions
B. Initial placement of orthodontic brackets
C. Intracanal endodontic treatment
D. Periodontal procedures

Forensic Identification utilizes______?

A. Lip prints
B. Lip Schutz bodies
C. Lip pits
D. Lip reading

Radiographs of a 40 years old female revealed radiolucent areas around several of her mandibular teeth, all which tested vital on the electric pulp tester, These area represent ?

A. Multiple granulomas
B. Periapical osteofibroses
C. Chronic periapical abscesses
D. Bone hypoplasias associated with opalescent teeth

Feature of acanthosis nigricans is______?

A. Insulinoma, obesity & cutaneous hypopigmentation
B. Insulin resistance, obesity, cutaneous hyperpigmentation
C. Thickening of spinous layer, insulin resistance, obesity
D. Thickening of spinous layer insulin resistance, lean

Chediak- Higashi syndrome is inherited as______?

A. X-linked dominant trait
B. Autosomal dominant
C. Autosomal recessive
D. X-linked recessive

Which of the following blood disease has a racial predilection ?

A. Purpura
B. Hemophilia
C. Polycythemia
D. Thalassemia

In Radionucide imaging the most useful radio pharmaceutical for skeletal imaging is_____?

A. Gallium 67 (67 Ga)
B. Technetium-99m (99m Tc-Sc)
C. Technetium-99m (99m Tc)
D. Technetium-99m linked to Methylene disphosonate (99m Tc-MDP)

The most striking haematological finding in agranulocytosis is____?

A. Decreased absolute neutrophil count
B. Increased absolute eosinophil count
C. Decreased absolute basophil count
D. Increased absolute monocyte count

Deficiency of all the three components of coagulation factor VIII result in_______?

A. Von willebrand’s disease
B. Haemophilia- A
C. Parahemophilia
D. Haemophilia – B

Which of the following is not true about thalassemia ?

A. Increased in number of globulin chain
B. There is erythrocyte fragility and hemolysis
C. Hypochromic microcytic anaemia is present
D. There is severe anaemia and thrombocytopenia

A hair on end appearance of the skull is seen in all of the following except______?

A. Thalassemia
B. Sickle anemia
C. Cooley’s anaemia
D. Pagets disease

Hemophilia is associated with______?

A. Normal bleeding time normal clotting time
B. Normal bleeding time prolonged clotting time
C. Prolonged bleeding time normal clotting time
D. Prolonged bleeding time prolonged clotting time:

One of the following syndrome is characterized by an esophageal web with resulting dysphagia, atrophic changes in the mucous membranes of the mouth and a hypochromic microcytic anemia_____________?

A. Marfan’s syndrome
B. Plummer-vinson
C. Meckels syndrome
D. sjogren’s syndrome

Oral manifestations of infectious mononucleosis is most commonly______?

A. Bluish red spots opposite maxillary molar
B. Pseudomembrane on gingiva
C. Pinpoint petechiae on the palate
D. Gingival hyperplasia

Chemotherapy can be successful during treatment of______?

A. Ameloblastoma
B. Leukemia
C. Fibrosarcoma
D. Basal cell carcinoma

Precancerous potential in plummer-vinson’s syndrome may be due to change in the epithelium like_____?

A. Atrophy
B. Hypertrophy
C. Acanthosis
D. All of the above

Which of the following is the most serious and life threatening blood dyscrasias caused with a drug______?

A. Aplastic anemia
B. Megaloblastic anemia
C. Thrombocytopenia
D. Hemolytic anemia

For extraction in a leukemic patient_______?

A. Consult physician
B. Obtain WBC count
C. Obtain platelet count
D. All of the above

Plummer – vinson syndrome___?

A. Is due to folic acid deficiency
B. common in males
C. Not associated with oral premalignancy
D. Strong association with post-cricoid carcinoma

Chronic granulocytic leukemia is due to_____?

A. Chromosomal deletion
B. Chromosomal mutation
C. Chromosomal translocation
D. None of the above

Common oral change seen with nutritional anemia is_____?

A. Enlarged tongue
B. Atrophic glossitis
C. Generalized osteolysis
D. Focal marrow expansion

Christmas disease is due to deficiency of_______?

A. Hageman Factor
B. Platelets
C. Plasma thromboplastin antecedent
D. Plasma thromboplastin component

Clinical features of infectious mononucleosis______?

A. Glandular involvement
B. Febrile
C. Palatine Petechiae
D. All of the above

Leucocytopenia is seen in______?

A. influenza
B. agranulocytosis
C. liver cirrhosis
D. All of the above

Which of the following is not associated with haemorrhage ?

A. Ecchymosis
B. Petechiae
C. Melanosis
D. Purpura

Which of the following agents is of value in the postoperative care of the hemophilic patient ?

A. Vitamin K
B. Monsel’s solution
C. Aminocaproic acid
D. Factor 8 cryoprecipitate

A young patient is hospitalized with petechiae of oral mucous membrane, marginal gingival hemorrhage and with a platelet count of 45,000/ cc The BT and Clot Retraction time are increased, RBC and TLC are normal He is suffering from_____?

A. Infectious mononucleosis
B. Thrombocytopenic purpura
C. Leukemia
D. Hemophilia

Hemophilia B is due to_____?

A. Factor VII deficiency
B. Factor IX deficiency
C. Platelet deficiency
D. it C deficiency

Which of the following disease is known as the Kissing disease______?

A. Acquired immunodeficiency syndrome (AIDS)
B. Infection mononucleosis
C. Primary syphilis
D. Recurrent apthous stomatitis

Neurological symptoms and premature graying of hair is associated with______?

A. Folic acid deficiency
B. Pernicious anaemia
C. Plummer-Vinson syndrome
D. Paterson-Kelly syndrome

Hair-on-end appearance in a skull roentgenogram is seen in______?

A. Fibrous dysplasia
B. Thalassemia
C. Garre’s Osteomyelitis
D. Pagets disease

The oral findings in erythroblastosis fetalis include______?

A. Dentinal dysplasia
B. Hypoplastic teeth
C. Pigmented teeth
D. All of the above

Commonest mode of inheritance of Von Willebrand’s disease is_____?

A. Codominant
B. Autosomal dominan
C. Autosomal recessive
D. X-Linked recessive

All the following are TRUE in Immune thrombocytopenic Purpura (ITP) EXCEPT_____?

A. Chronic ITP commonly occur in adult women
B. Associated with normal bleeding time
C. Prothrombin Time (PT) & Partial Thromboplastin Time (PTT) are normal
D. Increased megakaryocytes in bone marrow

The most reliable criteria in Gustafson’s method of identification is_____?

A. Cementum apposition
B. Transparency of root
C. Attrition
D. Root resorption

The most common coagulation disorders haemophilia A and von Willebrand’s disease are due to_______?

A. Factor IX deficiency
B. Vitamin K deficiency
C. Factor X deficiency
D. Factor VIII deficiency

Cooley’s anemia is also known as_____?

A. Erythroblastosis fetalis
B. Aplatic anemia
C. Thalassemia
D. Pernicious anemia

Erythroblastosis fetalis can be prevented if the mother is injected at parturition, with an antibody called_______?

A. Blocking antibody
B. Rh (D) immunoglobulin
C. Antilymphocyte globulin
D. Antithymocyte serum

A patient on warfarin sodium following myocardial infarction reports for an oral surgical procedure which one of the following laboratory tests should be preferred to ascertain the fitness______?

A. Prothrombin time
B. Tourniquet time
C. Clotting time
D. Bleeding time

Aplastic anaemia is common with_______?

A. Chloramphenicol
B. Cephalosporin
C. Tetracycline
D. Penicillin

megaloblastic anaemia occurs due to______?

A. Iron deficiency
B. Folate deficiency
C. Vitamin C deficiency
D. Protein deficiency

Virus responsible for infectious mononucleosis is______?

A. RNA paramyxo virus
B. Varicella zoster virus
C. Epstein Barr virus
D. Coxsackie virus A 16

Infectious mononucleosis has______?

A. Multiple draining sinuses
B. Ulcers which bruise easily
C. Palatal perforation
D. Alveolar bone loss

Monospot test is used to diagnose_____?

A. Pernicious anemia
B. Sickle cell anemia
C. Infectious mononucleosis
D. Leukemia

All of the following statements about acute leukemia in children are true except_____?

A. It characteristically causes gross gingival swelling
B. It may be manifested by mucosal pallor
C. It can cause abvious prupura
D. It is usually of the lymphoblastic variety

Bleeding joints is a characteristic feature of______?

A. Vit – C deficiency
B. Heamophilia
C. Vit – K deficiency
D. Thrombocytopenia

Hypopigmentation, gray streaks of hair, degranulation defect of neutrophils and neuropathy are seen in______?

A. alukemic leukemia
B. chronic granulocytic leukemia
C. lazy leukocyte syndrome
D. Chediak higashi syndrome

A boy complains of bleeding gums, swollen, joints with hemorrhage into joints His paternal and maternal uncle complains of same problem It is due to deficiency of factor________?

A. VIII
B. IX
C. X
D. VI

Petechial hemorrhage is seen in______?

A. cyclic neutropenia
B. agranulocytosis
C. pernicious anemia
D. thrombocytopenic purpura

Bleeding time is prolonged in______?

A. Haemophilia
B. Von Willebrand’s disease
C. Henoch Schenolein purpura
D. Telangiectasia

Necrotising ragged ulceration with no apparent inflammatory response is indicative of______?

A. Leucocytosis
B. Polycythemia vera
C. Sickle cell anemia
D. Agranulocytosis

Pernicious anemia is_____?

A. insufficient production of red cells
B. Improper maturation of red cells
C. can be correct by tablets of folic acid alone
D. can be correct by iron supplement

Pinpoint hemorrhages of < 1cm diameter are known as_____?

A. Petechiae
B. Ecchymoses
C. Purpura
D. Pustules

Paul Bunnell test is positive in______?

A. Infectious mononucleosis
B. Multiple myeloma
C. Malignant nerves
D. Rubella

To prevent excessive bleeding during surgery a patient with hemophilia A may be given_______?

A. Whole blood
B. Fresh frozen plasma
C. Factor VIII concentrate
D. Factor IX concentrate

Which of the following is seen in idiopathic thrombocytopenic purpura______?

A. Thrombocytosis
B. Increased prothrombin time
C. Increased bleeding time
D. Increased clotting time

Which of the following is not a finding in classical hemophilia (hemophilia A)___ ____?

A. Bleeding into soft tissues, muscles and joints
B. Decreased factor VIII
C. Increase prothrombin Time
D. Increase Partial thromboplastin Time

The red blood cells in beta thalassemia are typically____?

A. Macrocytic and normochromic
B. Microcytic and Hypochromic
C. Normocytic and hypochromic
D. Normocytic and normochromic

ll of the following statements about idiopathic thrombocytopenic purpura are true EXCEPT________?

A. It is associated with palatelet-specific auto-antibodies
B. It caused a prolonged bleeding time
C. It is often controlled by immunosuppressive treatment
D. It causes more prolonged hemorrhage than hemophilia

A bacterial disease with oral manifestations is________?

A. Herpes
B. measles
C. Diphtheria
D. leishmaniasis

Syphilis becomes seropositive in______?

A. Chancre (primary syphilis)
B. Muco patches (secondary syphilis)
C. Gumma (tertiary syphilis)
D. Congenital syphilis

Multinucleated giant cells are least likely to be found in which of the following disorders ?

A. Blastomycosis
B. Cat scratch fever
C. Sarcoidosis
D. Streptococcus pneumonia

Oral ulcers that are painless are associated with_____?

A. Secondary herpes
B. Primary syphilis
C. Tuberculosis
D. Primary herpes

Complications of Vincents angina is______?

A. Actinomycosis
B. Noma
C. Systemic candidiasis
D. Cellulites

Hansen’s disease is another name of_______?

A. Leprosy
B. Tuberculosis
C. Diabetes
D. Lichen planus

Hard swelling at the angle of mandible with numerous draining sinuses is most likely_____?

A. Actinomycosis
B. Ludwig’s angina
C. Mucormycosis
D. Cellulitis

Oral lesions are rarely seen in______?

A. AIDS
B. Tuberculosis
C. Syphilis
D. Leukemia

The oral lesion of syphilis that is highly infective is a______?

A. Gumma
B. Koplik spot
C. Mucous patcl
D. Tabes dorsalis

Strawberry Tongue is associated with______?

A. syphilis
B. measles
C. scarlet fever
D. typhoid

Which of the following oral structures are not effected in leprosy______?

A. Gingiva
B. Tongue
C. Hard palate
D. Soft palate

All of the following can give rise to membrane on the pharynx except______?

A. Staphylococcus aureus
B. Corynebacterium
C. Candida
D. Vincent’s angina

The causative agent for “Oculoglandular syndrome of parinoud” is______?

A. Arachnia propionica
B. Bartonella henselae
C. Bifidobacterium dentium
D. Mycobacterium laprae

Oral ulcers which occurs in groups, persist for about 6 weeks and leave scars on healing are______?

A. Recurrent aphthous major
B. Recurrent aphthous minor
C. Recurrent herpetiform ulcers
D. Acute herpetic gingivostomatitis

+ve pathergy test is seen in______?

A. Sarcoidosis
B. Histoplasmosis
C. Candidiasis
D. Behcet’s disease

The first consideration in the differential diagnosis of a painless palatal perforation would be_____?

A. Syphilis
B. Histoplasmosis
C. Scrofuloderma
D. Actinomycosis

Recurrent aphthae resemble recurrent herpes in that_______?

A. symptoms are similar
B. life long immunity results
C. vesicles occur with both diseases
D. intranuclear inclusion bodies are present

Kissing disease is also known as____?

A. Scarlet fever
B. Acute herpetic gingivostomatitis
C. Glandular fever
D. Rubella

The best laboratory test to use in the diagnosis of lupus vulgaris in the oral cavity is_______?

A. Bacterial smear
B. Blood studies
C. Biopsy
D. Blood chemistry

Lock jaw indicates_______?

A. Ankylosis
B. Spasm of masseter mucle
C. Auspitz’s sign
D. Nikolsky’s sign

Canker sore’ is otherwise commonly known as______?

A. Recurrent apthous ulcer
B. Recurrent herpetic gingivitis
C. Acute necrotizing ulcerative gingivitis
D. Recurrent herpes labialis

The explosive and widespread form of secondary syphilis in immuno compromised individual is known as_______?

A. Condylomata lata
B. Mucous patches
C. Lues maligna
D. lupus vulgaris

Crohn’s disease_______?

A. has oral ulcerations similar to major apthous ulceration
B. is a self limiting lesion
C. is commonly seen among Indian population
D. is usually treated with Erythromycin

Kveim Slitzbach test is usuful in the diagnosis of______?

A. Tuberculosis
B. Sarcoidosis
C. Leprosy
D. Cat scratch disease

A patient has asymptomatic painless burrowing ulcer in the palate, He might be suffering from______?

A. Syphilis
B. Tuberculosis
C. Histoplasmosis
D. Actinomycosis

Apthous like ulcers are seen in_______________?

A. Bechets syndrome
B. Sweet syndrome
C. PFAPA (periodic fever acute pharyngitis apthous stomatitis)
D. All of the above

Which of the following is a painfull ulcer ?

A. syphilitic ulcer
B. trophic ulcer
C. oral chancre
D. All of the above

Primary antibody deficiencies are characterized by_____?

A. Recurrent allergic reactions
B. Recurrent bacterial infections
C. Implicit allergic reactions
D. implicit bacterial infections

Oral lesions of secondary syphilis includes all EXCEPT_____?

A. snail track ulcers
B. Mucous patches
C. chancre of tongue
D. Hutchinson’s wart

Botryomycosis is a_______ disease?

A. Fungal
B. Bacterial
C. Viral
D. Parasitic

Tuberculous ulcer of oral cavity is usually_____?

A. painless
B. painful
C. itching
D. Asymptomatic

The oral lesion called mucous patches is usually multiple grayish white plaque associated with______?

A. Pain
B. No pain
C. Itching
D. Burning sensation

Early tongue changes seen in scarlet fever is____?

A. Rasbery tongue
B. Strawberry tongue
C. Auspitz’s sign
D. Nikolsky’s sign

Most common site of tuberculous lesion in the oral cavity is_____?

A. Buccal mucosa
B. Lips
C. Tongue
D. Palate

Which of the following ulcers are characteristically painless_______?

A. Trauma
B. Tuberculosis
C. Primary syphilis
D. Herpes

Secondary syphilis occurs after______?

A. 6 weeks
B. 9 weeks
C. 13 weeks
D. 1 weeks

Noma is_____?

A. pyogenic granuloma
B. fibrous dysplasia
C. vincent’s disease
D. ancrum oris

Trismus is due to infection by____?

A. Staphylococci
B. Streptococci
C. Diphtheria
D. Clostridium Tetani

Splitted papule at corner of mouth____?

A. Recurrent herpes labialis
B. Recurrent herpetic stomatitis
C. Increased vertical dimension
D. Secondary syphilis

Clinical features of which of the following include conjunctivitis, urethritis muco-cutaneous lesions and arthritis ?

A. Behcet’s syndrome
B. Hodgkin’s disease
C. Grinspan syndrome
D. Ehler Danlos syndrome

Actinomycosis is a______?

A. Bacterial infection
B. Fungal infection
C. Viral infection
D. Parasitic infection

Gumma occurs in______?

A. Primary stage of syphilis
B. Secondary stage of syphilis
C. Tertiary stage of syphilis
D. Primary tuberculosis

Hutchinson’s triad includes____?

A. hypoplasia of incisor and molar, VIII nerve motor paralysis and peg laterals.
B. Hypoplasia of incisor and molar, V nerve motor paralysis interstitial keratitis
C. Hypoplasia of incisor and molar, VIII nerve deafness and interstitial keratitis
D. None of the above

Lesions of oral tuberculosis are associated with all the following except_______?

A. Central caseation
B. Hyaline degeneration
C. Giant cells in the center
D. Presence of epithelioid cells

Spasm of masseter muscles occur in______?

A. Tetanus
B. Trigeminal neuraligia
C. Tuberculous meningitis
D. Rabies

Which disease are infections but not communicable ?

A. Measles
B. Mumps
C. Scarlet fever
D. Tetanus

Cancerous involvement is seen with_____?

A. Chancre
B. Mucous patch
C. Syphilitic glossitis
D. Gumma

The bacteria observed as a causative organism in case of Recurrent apthous ulcers is ______?

A. Streptococcus sangius
B. Streptococcus mutans
C. Borellia vincentii
D. Staphylococci albus

Rigidity of facial muscles Risus sardonicus is associated with_______?

A. Tetany
B. Tetanus
C. Leprosy
D. Actinomycosis

A case presenting with a gray coloured pseudomembrane whose removal is difficult and painful, can be_______?

A. ANUG
B. Diphtheritic lesion
C. Secondary stage of syphilis
D. Desquamative gingivitis

Sulfur granules in pus are seen in______?

A. Candida albicans
B. Actinomyces israelii
C. Nocarda braziliances
D. Histoplasma capsulatum

Leutic glossitis is an intra-oral manifestation caused by______?

A. Mycobacterium tuberulosis
B. Actinomycosis
C. Trepenoma pallidium
D. Streptococcus

Which of the following is Not characteristic of congenital Syphilis ?

A. Ghon complex
B. Interstitial keratitis
C. Mulberry molars
D. Notched incisors

Latest Posts

Scroll to Top